HESI Exit Exam

¡Supera tus tareas y exámenes ahora con Quizwiz!

A male client with COPD smokes two packs of cigarettes per day and is admitted to the hospital for a respiratory infection. He complains that he has trouble controlling respiratory distress at home when using his rescue inhaler. Which comment from the client indicates to the nurse that he is not using his inhaler properly?

"After I squeeze the inhaler and swallow, I always feel a slight wave of nausea, bit it goes away".

An infant is receiving penicillin G procaine 220,000 units IM. The drug is supplied as 600,000 units/ml. How many ml should the nurse administer? (Enter numeric value only. If rounding is required, round to the nearest tenth)

0.4. Rationale: Calculate using the formula, desired dose (220,000 units) over dose on hand (600,000 units) x the volume of the available dose (1 ml). 220,000 / 600,000 x 1 ml = 0.36 = 0.4 ml.

The nurse is using a straight urinary catheter kit to collect a sterile urine specimen from a female client. After positioning am prepping this client, rank the actions in the sequence they should be implemented. (Place to first action on the top on the last action on the bottom.)

1. Open the sterile catheter kit close to the client's perineum. 2. Don sterile gloves and prepare to sterile field. 3. Cleanse the urinary meatus using the solution, swabs, and forceps provided. 4. Place distal end of the catheter in sterile specimen cup and insert catheter into meatus. Rationale: First the kit should be open near the clients to minimize the risk of contamination during the collection of the sterile specimen. Once the kit is opened, sterile gloves should be donned to prepare the sterile field. Then the clients' meatus should be cleansed, and the catheter inserted while to distal end of the catheter drains urine into the sterile specimen cup or receptacle.

Nurses working on a surgical unit are concerned about the physicians treatment of clients during invasive procedures, such as dressing changes and insertion of IV lines. Clients are often crying during the procedures, and the physician is usually unconcerned or annoyed by the client's response. To resolve this problem, what actions should the nurses take? (Arrange from the first action on the top of the list on the bottom)

1. Talk to the physician as a group in a non-confrontational manner. 2. Document concerns and report them to the charge nurse. 3. Submit a written report to the director of nursing. 4. Contact the hospital's chief of medical services. 5. File a formal complaint with the state medical board. Rational: nurses have both an ethical and legal responsibility to advocate for clients' physical and emotional safety. Talking with the physician in a non-confrontational manner is the first step in conflict resolution. If this is not effective, the organizational chain of ineffective, a formal complaint with the state medical board should be implemented.

The healthcare provider prescribes potassium chloride 25 mEq in 500 ml D_5W to infuse over 6 hours. The available 20 ml vial of potassium chloride is labeled, "10 mEq/5ml." how many ml of potassium chloride should the nurse add the IV fluid? (Enter numeric value only. If is rounding is required, round to the nearest tenth.)

12.5 Rationale: Using the formula D / H X Q: 25 mEq / 10 mEq x 5ml= 12.5ml

The nurse is preparing a heparin bolus dose of 80 units/kg for a client who weighs 220 pounds. Heparin sodium injection, USP is available in a 3o ml multidose vial with the concentration of 1,000 USP units/ml. how many ml of heparin should the nurse administer? (Enter numeric value only)

8 Calculate the client's weigh in kg: 220 pounds divides by 2.2 pounds/kg 100 kg Calculate the client's dose, 80 units x 100 kg 8,000 units Use the formula, D / H X Q 8,000 units / 1,000 units x 1ml = 8

An adult client experiences a gasoline tank fire when riding a motorcycle and is admitted to the emergency department (ED) with full thickness burns to all surfaces of both lower extremities. What percentage of body surface area should the nurse document in the electronic medical record (EMR)?

9 % 18 % 36 % 45 % Rational: according to the rule of nines, the anterior and posterior surfaces of one lower extremity is designated as 18 %of total body surface area (TBSA), so both extremities equals 36% TBSA, other options are incorrect.

The charge nurse is planning for the shift and has a registered nurse (RN) and a practical nurse (PN) on the team. Which client should the charge nurse assign to the RN?

A 30 year old depressed client who admits to suicide ideation. Rational: A client who is suicidal requires psychological assessment, therapeutic communication and knowledge beyond the educational level of a practical nurse (RN). Other clients could be cared for by the PN or the UAP, with supervision by the registered nurse.

Which client should the nurse assess frequently because of the risk for overflow incontinence?

A client Who is confused and frequently forgets to go to the bathroom

What information should the nurse provide? (Select all that apply.)

A client must be willing to accept palliative care, not curative care. The healthcare provider must project that the client has 6 months or less to live.

A nurse working on an endocrine unit should see which client first?

A client taking corticosteroids who has become disoriented in the last two hours. Rational: meeting the client's need for safety is a priority intervention. Mania and psychosis can occur during corticosteroids therapy, places the client at risk for injury, so the patient taking corticosteroids should be seen first.

The nurse working in the psychiatric clinic has phone messages from several clients. Which call should the nurse return first?

A family member of a client with dementia who has been missing for five hours

A nurse with 10 years experience working in the emergency room is reassigned to the perinatal unit to work an 8 hour shift. Which client is best to assign to this nurse?

A mother with an infected episiotomy

A client is admitted to the hospital after experiencing a brain attack, commonly referred to as a stroke or cerebral vascular accident (CVA). The nurse should request a referral for speech therapy if the client exhibits which finding?

Abnormal responses for cranial nerves I and II Persistent coughing while drinking Unilateral facial drooping Inappropriate or exaggerated mood swings

After several hours of non-productive coughing, a client presents to the emergency room complaining of chest tightness and shortness of breath. History includes end stage chronic obstructive pulmonary disease (COPD) and diabetes mellitus. While completing the pulmonary assessment, the nurse hears wheezing and poor air movement bilaterally. Which actions should the nurse implement? (Select all that apply.)

Administer PRN nebulizer treatment. Obtain 12 lead electrocardiogram. Monitor continuous oxygen saturation.

An older adult resident of a long-term care facility has a 5-year history of hypertension. The client has a headache and rate the pain 5 on a pain scale 0 to 10. The client's blood pressure is currently 142/89. Which interventions should the nurse implement? (Select all that apply)

Administer a daily dose of lisinopril as scheduled. Provide a PRN dose of acetaminophen for headache. Rational: the client' routinely scheduled medication, lisinopril, is an antihypertensive medication and should be administered as scheduled to maintain the client's blood pressure. A PRN dose of acetaminophen should be given for the client's headache. The other options are not indicated for this situation.

While making rounds, the charge nurse notices that a young adult client with asthma who was admitted yesterday is sitting on the side of the bed and leaning over the bed-side-table. The client is currently receiving at 2 litters/minute via nasal cannula. The client is wheezing and is using pursed-lip breathing. Which intervention should the nurse implement?

Administer a nebulizer Treatment. Rationale: The client needs an immediate medicated nebulizer treatment. Sitting in an upright position with head and arms resting on the over-bed table is an ideal position to promote breathing because it promotes lung expansion. Other actions me be accurate but not yet indicated.

The nurse caring for a 3-month-old boy one day after a pylorotomy notices that the infant is restless, is exhibiting facial grimaces, and is drawing his knees to his chest. What action should the nurse take?

Administer a prescribed analgesia for pain.

A 6 -years-old who has asthma is demonstrating a prolonged expiratory phase and wheezing, and has 35% personal best peak expiratory flow rate (PEFR). Based on these finding, which action should the nurse implement first?

Administer a prescribed bronchodilator. Rationale: If the PEFR is below 50% in as asthmatic child, there is severe narrowing of the airway, and a bronchodilator should be administered immediately. Be should be implemented after A. C will not alleviate the symptoms and D is not a priority.

A 13 years-old client with non-union of a comminuted fracture of the tibia is admitted with osteomyelitis. The healthcare provider collects home aspirate specimens for culture and sensitivity and applies a cast to the adolescent's lower leg. What action should the nurse implement next?

Administer antiemetic agents Bivalve the cast for distal compromise Provide high- calorie, high-protein diet Begin parenteral antibiotic therapy Rationale: The standard of treatment for osteomyelitis is antibiotic therapy and immobilization. After bond and blood aspirate specimens are obtained for culture and sensitivity, the nurse should initiate parenteral antibiotics as prescribed.

A client experiencing withdrawal from the benzodiazepines alprazolam (Xanax) is demonstrating severe agitation and tremors. What is the best initial nursing action?

Administer naloxone (Narcan) per PNR protocol Initiate seizure precautions Obtain a serum drug screen Instruct the family about withdrawal symptoms. Rationale: Withdrawal of CNS depressants, such as Xanax, results in rebound over-excitation of the CNS. Since the client exhibiting tremors, the nurse should anticipate seizure activity and protect the client.

A client with a history of chronic pain requests a nonopioid analgesic. The client is alert but has difficulty describing the exact nature and location of the pain to the nurse. What action should the nurse implement next?

Administer the analgesic as requested

A client with a history of chronic pain requests a nonopioid analgesic. The client is alert but has difficulty describing the exact nature and location of the pain to the nurse. Which action should the nurse implement next?

Administer the analgesic as requested Rationale: Chronic pain may be difficult to describe but should be treated with analgesics as indicated.

A client receives a new prescription for simvastatin (Zocor) 5 mg PO daily at bedtime. What action should the nurse take?

Administer the medication as prescribed with a glass of water

The healthcare provider changes a client's medication prescription from IV to PO administration and double the dose. The nurse notes in the drug guide that the prescribed medication, when given orally, has a high first-pass effect and reduce bioavailability. What action should the nurse implement?

Administer the medication via the oral route as prescribed

A client diagnosed with calcium kidney stones has a history of gout. A new prescription for aluminum hydroxide (Amphogel) is scheduled to begin at 0730. Which client medication should the nurse bring to the healthcare provider's attention?

Allopurinol (Zyloprim) Aspirin, low dose Furosemide (lasix) Enalapril (vasote)

A female nurse who took drugs from the unit for personal use was temporarily released from duty. After completion of mandatory counseling, the nurse has asked administration to allow her to return to work. When the nurse administrator approaches the charge nurse with the impaired nurse request, which action is best for the charge nurse to take?

Allow the impaired nurse to return to work and monitor medication administration. Rationale: provides essential monitoring and helps ensure nurse compliance and promote client safety.

The charge nurse is making assignment on a psychiatric unit for a practical nurse (PN) and newly license register nurse (RN). Which client should be assigned to the RN?

An adult female who has been depress for the past several month and denies suicidal ideations. A middle-age male who is in depressive phase on bipolar disease and is receiving Lithium. A young male with schizophrenia who said voices is telling him to kill his psychiatric. An elderly male who tell the staff and other client that he is superman and he can fly. Rationale: The RN should deal with the client with command hallucinations and these can be very dangerous if the client's acts on the commands, especially if the command is a homicidal in nature. Other client present low safety risk.

In making client care assignment, which client is best to assign to the practical nurse (PN) working on the unit with the nurse?

An immobile client receiving low molecular weight heparin q 12 h. Rationale: A describe the most stable client. The other ones are at high risk for bleeding problems and require the assessment skills.

After receiving the Braden scale findings of residents at a long-term facility, the charge nurse should to tell the unlicensed assistive personnel (UAP) to prioritize the skin care for which client?

An older man whose sheets are damped each time he is turned. Rational: a Braden score of less than 18 indicates a risk for skin breakdown, and clients with such score require intensive nursing care. Constant moisture places the client at a high risk for skin breakdown, and interventions should be implemented to pull moisture away from the client's skin. Other options may be risk factors but do not have as high a risk as constant exposure to moisture

A 16-year-old adolescent with meningococcal meningitis is receiving a continuous IV infusion of penicillin G, which is prescribed as 20 million units in a total volume of 2 liters of normal saline every 24 hr. The pharmacy delivers 10 million units/ liters of normal saline. How many ml/hr should the nurse program the infusion pump? (Enter numeric value only. If rounding is required, round to the nearest whole number.)

Answer 83 Rationale: 1000 ml-----12hr. Xml ---------1hr. 1000/12 = 83.33 = 83.

A client is receiving and oral antibiotic suspension labeled 250 mg/2ml. The healthcare provider prescribes 200mg every 6 hours. How many ml should the nurse administer at each dose? (Enter numerical value only. If rounding is required, round to the nearest tenth)

Answer: 1.6. Rational: using the formula D/H x Q 200mg/250 mg x 2ml = 200/250 = 1.6 ml.

The nurse uses the parkland formula (4ml x kg x total body surface area = 24 hours fluid replacement) to calculate the 24-hours IV fluid replacement for a client with 40% burns who weighs 76kg. How many ml should the client receive? (Enter numeric value only.)

Answer: 12160 Rationale: 4ml x 67kg x 40 (bsa) =12,160 ml

The nurse assesses a client with new onset diarrhea. It is most important for the nurse to question the client about recent use of which type of medication?

Antibiotics Anticoagulants Antihypertensive Anticholinergics

In assessing an older female client with complication associated with chronic obstructive pulmonary disease (COPD), the nurse notices a change in the client's appearance. Her face appears tense and she begs the nurse not to leave her alone. Her pulse rate is 100, and respirations are 26 per min. What is the primary nursing diagnosis?

Anxiety related to fear of suffocation.

After repositioning an immobile client, the nurse observes an area of hyperemia. To assess for blanching, what action should the nurse take?

Apply light pressure over the area.

When assessing and adult male who presents as the community health clinic with a history of hypertension, the nurse note that he has 2+ pitting edema in both ankles. He also has a history of gastroesophageal reflex disease (GERD) and depression. Which intervention is the most important for the nurse to implement?

Arrange to transport the client to the hospital Instruct the client to keep a food journal, including portions size. Review the client's use of over the counter (OTC) medications. Reinforce the importance of keeping the feet elevated. Rationale: Sodium is used in several types of OTC medications. Including antacids, which the client may be using to treat his GERD. Further evaluation is need it to determine the need for hospitalization (A) A food journal (B) may help over, but dietary modifications are needed now since edema is present. (C) May relieve dependent edema, but not treat the underlying etiology.

A client who had a percutaneous transluminal coronary angioplasty (PTCA) two weeks ago returns to the clinic for a follow up visit. The client has a postoperative ejection fraction ejection fraction of 30%. Today the client has lungs which are clear, +1 pedal edema, and a 5 pound weight gain. Which intervention the nurse implement?

Arrange transport for admission to the hospital. Insert saline lock for IV diuretic therapy. Assess compliance with routine prescriptions. Instruct the client to monitor daily caloric intake. Rationale: Fluid retention may be a sign that the client is not taking the medication as prescribed or that the prescriptions may need adjustment to manage cardiac function post-PTCA (normal ejection fraction range is 50 to 75%)

The development of atherosclerosis is a process of sequential events. Arrange the pathophysiological events in orders of occurrence. (Place the first event on top and the last on the bottom)

Arterial endothelium injury causes inflammation Macrophages consume low density lipoprotein (LDL), creating foam cells Foam cells release growth factors for smooth muscle cells Smooth muscle grows over fatty streaks creating fibrous plaques Vessel narrowing results in ischemia

When should intimate partner violence (IPV) screening occur?

As a routine part of each healthcare encounter. Rationale: Universal screening for IPV is a vital means to identify victims of abuse in relationship. The suspicious of different clinicians vary greatly, so screening would not be implemented consistently. The client should be screened regardless of the presence of injury. Although history of abuse is difficult to confirm, screening should occur regardless, and this incident may know may be initial case of abuse.

A client is scheduled to receive an IW dose of ondansetron (Zofran) eight hours after receiving chemotherapy. The client has saline lock and is sleeping quietly without any restlessness. The nurse caring for the client is not certified in chemotherapy administration. What action should the nurse take?

Ask a chemotherapy-certified nurse to administer the Zofran Administer the Zofran after flushing the saline lock with saline Hold the scheduled dose of Zofran until the client awakens Awaken the client to assess the need for administration of the Zofran. Rationale: Zofran is an antiemetic administered before and after chemotherapy to prevent vomiting. The nurse should administer the antiemetic using the accepter technique for IV administration via saline lock. Zofran is not a chemotherapy drug and does not need to be administered by a chemotherapy- certified nurse.

An adult client present to the clinic with large draining ulcers on both lower legs that are characteristics of Kaposi's sarcoma lesions. The client is accompanied by two family member. Which action should the nurse take?

Ask family member to wear gloves when touching the patient Send family to the waiting area while the client's history is taking Obtain a blood sample to determine is the client is HIV positive Complete the head to toes assessment to identify other sign of HIV Rationale: To protect the client privacy, the family member should be asked to wait outside while the client's history is take. Gloves should be worn when touching the client's body fluids if the client is HIV positive and these lesion are actually Kaposi sarcoma lesion. HIV testing cannot legally be done without the client explicit permission. A further assessment can be implemented after the family left the room.

An unlicensed assistive personnel (UAP) reports that a client's right hand and fingers spasms when taking the blood pressure using the same arm. After confirming the presence of spams what action should the nurse take?

Ask the UAP to take the blood pressure in the other arm Tell the UAP to use a different sphygmomanometer. Review the client's serum calcium level Administer PRN antianxiety medication. Rationale: Trousseau's sign is indicated by spasms in the distal portion of an extremity that is being used to measure blood pressure and is caused by hypocalcemia (normal level 9.0-10.5 mg/dl, so C should be implemented.

A male client with cancer is admired to the oncology unit and tells the nurse that he is in the hospital for palliative care measures. The nurse notes that the client's admission prescription include radiation therapy. What action should the nurse implement?

Ask the client about his expected goals for the hospitalization. Rationale: Palliative care measures provide relief or control of symptoms, so it is important for the nurse to determine the client's goals for symptom control while receiving treatment in the hospital. Although home care is available the client may not be legible for palliative care at home. Radiation therapy is an effective positive care measure used to manage symptoms and would be appropriate unless the radiation conflicts with the client goals.

While teaching a young male adult to use an inhaler for his newly diagnosed asthma, the client stares into the distance and appears to be concentrating on something other than the lesson the nurse is presenting. What action should the nurse take?

Ask the client what he is thinking about at his time.

A preschool-aged boy is admitted to the pediatric unit following successful resuscitation from a near- drowning incident. While providing care to child, the nurse begins talking with his preadolescent brother who rescued the child from the swimming pool and initiated resuscitation. The nurse notices the older boy becomes withdrawn when asked about what happened. What action should the nurse take?

Ask the older brother how he felt during the incident.

An older male client is admitted with the medical diagnosis of possible cerebral vascular accident (CVA). He has facial paralysis and cannot move his left side. When entering the room, the nurse finds the client's wife tearful and trying unsuccessfully to give him a drink of water. What action should the nurse take?

Ask the wife to stop and assess the client's swallowing reflex

Which intervention should the nurse implement during the administration of vesicant chemotherapeutic agent via an IV site in the client's arm?

Assess IV site frequently for signs of extravasation

During change of shift, the nurse reports that a male client who had abdominal surgery yesterday increasingly confused and disoriented during the night. He wandered into other clients rooms, saying that there are men in his room trying to hurt him. Because of continuing disorientation and the client's multiple attempts to get of bed, soft restrains were applied at 0400. In what order should the nurse who is receiving report implement these interventions? (Arrange from first action on top to last on the bottom).

Assess the client's skin and circulation for impairment related to the restrains Evaluate the client's mentation to determine need to continue the restrains Assign unlicensed assistive personnel to remove restrains and remain with client Contact the client's surgeon and primary healthcare provider

An older male comes to the clinic with a family member. When the nurse attempts to take the client's health history, he does not respond to questions in a clear manner. What action should the nurse implement first?

Assess the surroundings for noise and distractions.

An adult who is 5 feet 5 inches (165.1 cm) tall and weighs 90 lb. (40.8 Kg) is admitted with a diagnosis of chronic anorexia. The client receives a regular diet for 2 days, and the client's medical records indicates that 100% of the diet provided has been consumed. However the client's weight on the third day morning after admission is 89 lb. (40.4 Kg). What action should the nurse implement?

Assign staff to monitor what the client eats. Rationale: clients with an eating disorder have an unhealthy obsession with food. The client's continued weight loss, despites indication that the client has consumed 100% of the diet, should raise questions about the client's intake of the food provided, so the client should be observed during meals to prevent hiding or throwing away food. Other options may be accurate but ineffective and unnecessary

An adult client with severe depression was admitted to the psychiatric unit yesterday evening. Although the client ran one year ago, his spouse states that the client no longer runs, bur sits and watches television most of the day. Which is most important for the nurse to include in this client's plan of care for today?

Assist client in identifying goals for the day. Rationale: clients with severe depression have low energy and benefit from structured activities because concentration is decreased. The client participate in care by identifying goals for the day is the most important intervention for the client's first day at the unit. Other options can be implemented over time, as the depression decreases

Which problem, noted in the client's history, is important for the nurse to be aware of prior to administration of a newly prescribed selective serotonin reuptake inhibitor (SSRI)?

Aural migraine headaches.

A male client, who is 24 hours postoperative for an exploratory laparotomy, complains that he is "starving" because he has had no "real food" since before the surgery. Prior to advancing his diet, which intervention should the nurse implement?

Auscultate bowel sounds in all four quadrants

The nurse identifies an electrolyte imbalance, an elevated pulse rate, and elevated BP for a client with chronic kidney disease. Which is the most important action for the nurse to take?

Auscultate for irregular heart rate. Rational: Chronic kidney failure (CKF) is a progressive, irreversible loss of kidney functions, decreasing glomerular filtration rate (GFR), and the kidney's inability to excrete metabolic waste products and water, resulting in fluid overload, elevated pulse, elevated BP and electrolytes imbalances. The most important action for the nurse to implement is to auscultate for irregular heart rate (D) due to the decreased excretion of potassium by the kidneys. (A, B, and C) are not as important as monitoring for fatal cardiac dysrhythmias related to hyperkalemia

The nurse should teach the client to observe which precaution while taking dronedarone?

Avoid grapefruits and its juice

The nurse is preparing an older client for discharge following cataract extraction. Which instruction should be include in the discharge teaching?

Avoid straining at stool, bending, or lifting heavy objects. Rationale: after cataract surgery, the client should avoid activities which increase pressure and place strain on the suture line.

A client with atrial fibrillation receives a new prescription for dabigatran. What instruction should the nurse include in this client's teaching plan?

Avoid use of nonsteroidal ant-inflammatory drugs (NSAID).

The nurse is evaluating the diet teaching of a client with hypertension. What dinner selection indicates that the client understands the dietary recommendation for hypertension?

Baked pork chop, applesauce, corn on the cob, 2% milk, and key-lime pie

An adult male reports the last time he received penicillin he developed a severe maculopapular rash all over his chest. What information should the nurse provide the client about future antibiotic prescriptions?

Be alert for possible cross-sensitivity to cephalosporin agents.

A client is receiving mesalamine 800 mg PO TID. Which assessment is most important for the nurse to perform to assess the effectiveness of the medication?

Bowel patterns Rationale: the client should be assessed for a change in bowel patterns to evaluate the effectiveness of this medication because Mesalamine is used to treat ulcerative colitis (a condition which causes swelling and sores in the lining of the colon [large intestine] and rectum) and also to maintain improvement of ulcerative colitis symptoms. Mesalamine is in a class of medications called anti-inflammatory agents. It works by stopping the body from producing a certain substance that may cause inflammation.

The mother of a child with cerebral palsy (CP) ask the nurse if her child's impaired movements will worsen as the child grows. Which response provides the best explanation?

Brain damage with CP is not progressive but does have a variable course

A child with heart failure is receiving the diuretic furosemide (Lasix) and has serum potassium level 3.0 mEq/L. Which assessment is most important for the nurse to obtain?

Cardiac rhythm and heart rate. Daily intake of foods rich in potassium. Hourly urinary output Thirst ad skin turgor.

The public nurse health received funding to initiate primary prevention program in the community. Which program the best fits the nurse's proposal?

Case management and screening for clients with HIV. Regional relocation center for earthquake victims Vitamin supplements for high-risk pregnant women. Lead screening for children in low-income housing. Rational: Primary prevention activities focus on health promotions and disease preventions, so vitamin for high-risk pregnant women provide adequate vitamin and mineral for fetal developmental.

A client who underwent an uncomplicated gastric bypass surgery is having difficult with diet management. What dietary instruction is most important for the nurse to explain to the client?

Chew food slowly and thoroughly before attempting to swallow Plan volume-controlled evenly-space meal thorough the day Sip fluid slowly with each meal and between meals Eliminate or reduce intake fatty and gas forming food Rationale: It is most important for the client to learn how to eat without damaging the surgical site and to keep the digestive system from dumping the food instead of digesting it. Eating volume-control and evenly-space meals thorough the day allows the client to fill full, avoid binging, and eliminate the possibility of eating too much one time. Chewing slowly and thoroughly helps prevent over eating by allowing a filling of fullness to occur. Taking sips, rather than large amounts of fluids keeps the stomach from overfilling and allow for adequate calories to be consumed. Gas forming foods and fatty foods should be avoiding to decrease risk of dumping syndrome and flatulence.

The charge nurse in a critical care unit is reviewing clients' conditions to determine who is stable enough to be transferred. Which client status report indicates readiness for transfer from the critical care unit to a medical unit?

Chronic liver failure with a hemoglobin of 10.1 and slight bilirubin elevation

A client with a lower respiratory tract infection receives a prescription for ciprofloxacin 500mg PO q 12 hours. When the client request an afternoon snack, which dietary choice should the nurse provide?

Cinnamon applesauce. Rational: Dairy products and calcium fortified dairy products decrease the absorption of ciprofloxacin. Cinnamon applesauce contains no calcium, so this is the best snack selection. Since other options contains calcium, these snack should be avoided by a client who is taking ciprofloxacin

The nurse plans to collect a 24- hour urine specimen for a creatinine clearance test. Which instruction should the nurse provide to the adult male client?

Clearance around the meatus, discard first portion of voiding, and collect the rest in a sterile bottle Urinate at specific time, discard the urine, and collect all subsequent urine during the next 24 hours. For the next 24 hours, notify the nurse when the bladder is full, and the nurse will collect catheterized specimens. Urinate immediately into a urinal, and the lab will collect specimen every 6 hours, for the next 24 hours. Rationale: Urinate at specific time, discard the urine, and collect all subsequent urine during the next 24 hours is the correct procedure for collecting 24-hour urine specimen. Discarding even one voided specimen invalidate the test.

A male client's laboratory results include a platelet count of 105,000/ mm3 Based on this finding the nurse should include which action in the client's plan of care?

Cluster care to conserve energy Initiate contact isolation Encourage him to use an electric razor Asses him for adventitious lung sounds Rationale: This client is at risk for bleeding based on his platelet count (normal 150,000 to 400,000/ mm3). Safe practices, such as using an electric razor for shaving, should be encouraged to reduce the risk of bleeding.

A male client is admitted for the removal of an internal fixation that was inserted for the fracture ankle. During the admission history, he tells the nurse he recently received vancomycin (vancomycin) for a methicillin-resistant Staphylococcus aureus (MRSA) wound infection. Which action should the nurse take? (Select all that apply.)

Collect multiple site screening culture for MRSA Call healthcare provider for a prescription for linezolid (Zyrovix) Place the client on contact transmission precautions Obtain sputum specimen for culture and sensitivity Continue to monitor for client sign of infection. Rationale: Until multi-site screening cultures come back negative (A), the client should be maintained on contact isolation(C) to minimize the risk for nosocomial infection. Linezolid (Zyvox), a broad spectrum anti-infectant, is not indicated, unless the client has an active skin structure infection cause by MRSA or multidrug- resistant strains (MDRSP) of Staphylococcus aureus. A sputum culture is not indicated9D) based on the client's history is a wound infection.

When evaluating a client's rectal bleeding, which findings should the nurse document?

Color characteristics of each stool.

To reduce staff nurse role ambiguity, which strategy should the nurse manager implemented?

Confirm that all the staff nurses are being assigned to equal number of clients. Review the staff nurse job description to ensure that it is clear, accurate, and recurrent. Assign each staff nurse a turn unit charge nurse on a regular, rotating basis. Analyze the amount of overtime needed by the nursing staff to complete assignments. Rationale: Role ambiguity occurs when there is inadequate explanation of job descriptions and assignedtasks, as well as the rapid technological changes that produce uncertainty and frustration. A and D may be implemented if the nurse manager is concerned about role overload, which is the inability to accomplish the tasks related to one's role. C is not related to ambiguity.

The nurse is caring for a client receiving continuous IV fluids through a single lumen central venous catheter (CVC). Based on the CVC care bundle, which action should be completed daily to reduce the risk for infection?

Confirm the necessity for continued use of the CVC.

A client is admitted with acute pancreatitis. The client admits to drinking a pint of bourbon daily. The nurse medicates the client for pain and monitors vital signs q 2 hours. Which finding should the nurse report immediately to the healthcare provider?

Confusion and tremors. Rationale: daily alcohol is the likely etiology for the client's pancreatitis. Abrupt cessation of alcohol can result in delirium tremens (DT) causing confusion and tremors, which can precipitate cardiovascular complications and should be reported immediately to avoid life-threatening complications. The other options are expected findings in those with liver dysfunction or pancreatitis, but do not require immediate action.

An older male client with type 2 diabetes mellitus reports that has experiences legs pain when walking short distances, and that the pain is relieved by rest. Which client behavior indicates an understanding of healthcare teaching to promote more effective arterial circulation?

Consistently applies TED hose before getting dressed in the morning. Frequently elevated legs thorough the day. Inspect the leg frequently for any irritation or skin breakdown Completely stop cigarette/ cigar smoking. Rationale: Stopping cigarette smoking helps to decrease vasoconstriction and improve arterial circulation to the extremity.

While in the medical records department, the nurse observes several old medical records with names visible in waste container. What action should the nurse implement?

Contact the medical records department supervisor.

A primigravida a 40-weeks gestation with preeclampsia is admitted after having a seizure in the hot tub at a midwife's birthing center. Based on documentation in the medical record, which action should the nurse implement? (Click on each chart tab for additional information. Please be sure to scroll to the bottom right corner of each tab to view all information contained in the client's medical record.)

Continue to monitor the client's blood pressure hourly.

The nurse is caring for a client who is entering the second stage of labor. Which action should the nurse implement first?

Convey to the client that birth is imminent.

Which action should the nurse implement with auscultating anterior breath sounds? (Place the first action on top and last action on the bottom)

Correct order: (PADD). 1. Place stethoscope in suprasternal area to auscultate for bronchial sounds. 2. Auscultate bronchovesicular sounds from side to side the first and second intercostal spaces. 3. Displace female breast tissue and apply stethoscope directly on chest wall to hear vesicular sounds. 4. Document normal breath sounds and location of adventitious breath sounds.

A client with osteoporosis related to long-term corticosteroid therapy receives a prescription for calcium carbonate. Which client's serum laboratory values requires intervention by the nurse?

Creatinine 4 mg/dl (354 micromol/L SI).

A client with a chronic health problem has difficulty ambulating short distance due to generalized weakness, but is able to bear weight on both legs. To assist with ambulation and provide the greatest stability, what assistive device is best for this client?

Crutches with 2 point gait. Crutches with 3 point gait. Crutches with 4 point gait. A quad cane

When assessing a 6-month old infant, the nurse determines that the anterior fontanel is bulging. In which situation would this finding be most significant?

Crying Straining on stool Vomiting Sitting upright. Rationale: The anterior fontanel closes at 9 months of age and may bulge when venous return is reduced from the head, but a bulging anterior fontanel is most significant if the infant is sitting up and may indicated an increase in cerebrospinal fluid. Activities that reduce venous return from the head, such as crying, a Valsalva maneuver, vomiting or a dependent position of the head, cause a normal transient increase in intracranial pressure.

While caring for a client's postoperative dressing, the nurse observes purulent drainage at the wound. Before reporting this finding to the healthcare provider, the nurse should review which of the client's laboratory values?

Culture for sensitive organisms. Rational: A client who has a postoperative dressing with purulent drainage from the wound is experiencing an infection. The nurse should review the client's laboratory culture for sensitive organisms (C) before reporting to the healthcare provider. (A, B and D) are not indicated at this time.

A client exposed to tuberculosis is scheduled to begin prophylactic treatment with isoniazid. Which information is most important for the nurse to note before administering the initial dose?

Current diagnosis of hepatitis B. Rationale: prophylactic treatment of tuberculosis with isoniazid is contraindicated for persons with liver disease because it may cause liver damage. The nurse should withhold the prescribed dose and contact the healthcare provider. Other options do not provide data indicating the need to question or withhold the prescribed treatment.

A client with hyperthyroidism is receiving propranolol (Inderal). Which finding indicates that the medication is having the desired effect?

Decrease in serum T4 levels Increase in blood pressure Decrease in pulse rate Goiter no longer palpable

An increased number of elderly persons are electing to undergo a new surgical procedure which cures glaucoma. What effect is the nurse likely to note as a result of this increases in glaucoma surgeries?

Decrease prevalence of glaucoma in the population.

The nurse is preparing to administer a histamine 2-receptor antagonist to a client with peptic ulcer disease. What is the primary purpose of this drug classification?

Decreases the amount of HCL secretion by the parietal cells in the stomach

An elderly male client is admitted to the mental health unit with a sudden onset of global disorientation and is continuously conversing with his mother, who died 50 years ago. The nurse reviews the multiple prescriptions he is currently taking and assesses his urine specimen, which is cloudy, dark yellow, and has foul odor. These findings suggest that his client is experiencing which condition?

Delirium Depression Dementia Psychotic episode

When administering an immunization in an adult client, the nurse palpates and administer the injection one inch below the acromion process into the center of the muscle mass. The nurse should document that the vaccine was administered at what site?

Deltoid. Rationale: The acromion process is a parameter identified for the deltoid site.

A client with hyperthyroidism is being treated with radioactive iodine (I-131). Which explanation should be included in preparing this client for this treatment?

Describe radioactive iodine as a tasteless, colorless medication administered by the healthcare provider

A male client with a long history of alcoholism is admitted because of mild confusion and fine motor tremors. He reports that he quit drinking alcohol and stopped smoking cigarettes one month ago after his brother died of lung cancer. Which intervention is most important for the nurses to include in the client's plan of care?

Determine client's level current blood alcohol level. Observe for changes in level of consciousness. Involve the client's family in healthcare decisions. Provide grief counseling for client and his family. Rationale: Based on the client's history of drinking, he may be exhibiting sing of hepatic involvement and encephalopathy. Changes in the client's level of consciousness should be monitored to determine if he able to maintain consciousness, so neurological assessment has the highest priority.

After a colon resection for colon cancer, a male client is moaning while being transferred to the Postanesthesia Care Unit (PACU). Which intervention should the nurse implement first?

Determine client's pulse, blood pressure, and respirations

The nurse is preparing an intravenous (IV) fluid infusion using an IV pump. Within 30 seconds of turning on the machine, the pump's alarm beeps "occlusion". What action should the nurse implement first?

Determine if the clamp on the IV tubing is released. Rational: When the pump immediately beeps, it is often because the IV tubing clamp is occluding the flow, so the clamp should be checked first to ensure that it is open. If the alarm is not eliminated after the tubing clamp is released, flushing the IV site with saline is a common practice to clean the needle or to identify resistance due to another source. Local signs of infiltration may indicate the need to select another vein, but the pump's beeping-this early in the procedure is likely due to a mechanical problem. If beeping continues after verifying that the clamp is released the placement or threading of the tubing through the pump should be verified.

A client with persistent low back pain has received a prescription for electronic stimulator (TENS) unit. After the nurse applies the electrodes and turns on the power, the client reports feeling a tingling sensation. How should the nurse respond?

Determine if the sensation feels uncomfortable. Rational: electronic stimulators, such as a transelectrical nerve stimulator (TENS) unit, have been found to be effective in reducing low back pain by "closing the gate" to pain stimuli. A tingling sensation should be felt when the power is turned on, and the nurse should assess whether the sensation is too strong, causing discomfort or muscle twitching. Decreasing the electrical signal may be indicated if the sensation is too strong. Other options are not necessary because the tingling sensation is expected.

A client arrives on the surgical floor after major abdominal surgery. What intervention should the nurse perform first?

Determine the client's vital sign.

A young boy who is in a chronic vegetative state and living at home is readmitted to the hospital with pneumonia and pressure ulcers. The mother insists that she is capable of caring for her son and which action should the nurse implement next?

Determine the mother's basic skill level in providing care. Rational: Although the mother states she is a capable caregiver, the client is manifesting disuse syndrome complications, and the mother's skill in providing basic care should be determined. Further assessment is needed before implementing other nursing actions.

A mother runs into the emergency department with s toddler in her arms and tells the nurse that her child got into some cleaning products. The child smells of chemicals on hands, face, and on the front of the child's clothes. After ensuring the airway is patent, what action should the nurse implement first?

Determine type of chemical exposure. Rational: once the type of chemical is determined, poison control should be called even if the chemical is unknown. If lavage is recommended by poison control, intubation and nasogastric tube may be needed as directed by poison control. Altered sensorium, such as lethargy, may occur if hydrocarbons are ingested.

Which problem reported by a client taking lovastatin requires the most immediate fallow up by the nurse?

Diarrhea and flatulence Abdominal cramps Muscle pain Altered taste Rationale: statins can cause rhabdomyolysis, a potentially fatal disease of skeletal muscle characterized by myoglobinuria and manifested with muscle pain, so this symptom should immediately be reported to the HCP.

An infant who is admitted for surgical repair of a ventricular septal defect (VSD) is irritable and diaphoretic with jugular vein distention. Which prescription should the nurse administer first?

Digoxin.

A female client with acute respiratory distress syndrome (ARDS) is chemically paralyzed and sedated while she is on as assist-control ventilator using 50% FIO2. Which assessment finding warrants immediate intervention by the nurse?

Diminished left lower lobe sounds Rationale: Diminished lobe sounds indicate collapsed alveoli or tension pneumothorax, which required immediate chest tube insertion to re-inflate the lung.

During a routine clinic visit, an older female adult tells the nurse that she is concerned that the flu season is coming soon, but is reluctant to obtain the vaccination. What action should the nurse take first?

Discuss the concerns expressed by the client about the vaccination. Rationale: the nurse should first address the concerns identified by the client, before taking other actions, such as obtaining information about past vaccinations, exposure to the flu, or reviewing the informed consent form.

The nurse assigned unlicensed assistive personnel (UAP) to apply antiembolism stockings to a client. The nurse and UAP enters the room, the nurse observes the stockings that were applying by the UAP. The UAP states that the client requested application of the stockings as seen on the picture, for increased comfort. What action should the nurse take?

Discussed effective use of the stockings with the client on UAP. Rational: antiembolism stockings are designed to fit securely and should be applied so that there are no bands of the fabric constricting venous return. The nurse should discuss the need for correct and effective use of the stockings with both the client and UAP to improve compliance. Other options do not correct the incorrect application of the stockings.

To prevent infection by auto contamination during the acute phase of recovery from multiple burns, which intervention is most important for the nurse to implement?

Dress each wound separately. Rational: each wound should be dressed separately using a new pair of sterile glove to avoid auto contamination (the transfer of microorganisms form one infected wound to a non-infected wound). The other choices do not prevent auto contamination.

A client with chronic alcoholism is admitted with a decreased serum magnesium level. Which snack option should the nurse recommend to this client?

Dry roasted almonds. Rational: alcoholism promotes inadequate food intake and gastrointestinal loss of magnesium include green leafy vegetables and nuts and seeds. Other snacks listed provide much lower amounts of magnesium per serving.

The nurse is planning a class for a group of clients with diabetes mellitus about blood glucose monitoring. In teaching the class as a whole, the nurse should emphasize the need to check glucose levels in which situation?

During acute illness

A woman just learned that she was infected with Heliobacter pylori. Based on this finding, which health promotion practice should the nurse suggest?

Encourage screening for a peptic ulcer

A female client is extremely anxious after being informed that her mammogram was abnormal and needs to be repeated. Client is tearful and tells the nurse her mother died of breast cancer. What action should the nurse take?

Encourage the client to continue expressing her fears and concerns. Rational: the nurse should show support for the client by encouraging her to continue expressing her concerns. A diagnosis has not yet been made, so it is too early to discuss treatment options. Other options dismiss the client's feelings or are premature given that the diagnosis is not yet made.

While receiving a male postoperative client's staples de nurse observe that the client's eyes are closed and his face and hands are clenched. The client states, "I just hate having staples removed". After acknowledgement the client's anxiety, what action should the nurse implement?

Encourage the client to continue verbalize his anxiety Attempt to distract the client with general conversation Explain the procedure in detail while removing the staples Reassure the client that this is a simple nursing procedure. Rational: Distract is an effective strategy when a client experience anxiety during an uncomfortable procedure. (A & D) increase the client's anxiety.

A resident of a long-term care facility, who has moderate dementia, is having difficulty eating in the dining room. The client becomes frustrated when dropping utensils on the floor and then refuses to eat. What action should the nurse implement?

Encourage the client to eat finger foods.

The nurse note a depressed female client has been more withdrawn and non-communicative during the past two weeks. Which intervention is most important to include in the updated plan of care for this client?

Encourage the client's family to visit more often Schedule a daily conference with the social worker Encourage the client to participate in group activities Engage the client in a non-threatening conversation. Rationale: Consistent attempts to draw the client into conversations which focus on non-threatening subjects can be an effective means of eliciting a response, thereby decreasing isolation behaviors. There is not sufficient data to support the effectiveness of A as an intervention for this client. Although B may be indicated, nursing interventions can also be used to treat this client. C is too threatening to this client.

During a Woman's Health fair, which assignment is the best for the Practical Nurse (PN) who is working with a register nurse (RN)

Encourage the woman at risk for cancer to obtain colonoscopy. Present a class of breast-self examination Prepare a woman for a bone density screening Explain the follow-up need it for a client with prehypertension. Rationale: A bone density screening is a fast, noninvasive screening test for osteoporosis that can be explained by the PN. There is no additional preparation needed (A) required a high level of communication skill to provide teaching and address the client's fear. (B) Requires a higher level of client teaching skill than responding to one client. (D) Requires higher level of knowledge and expertise to provide needed teaching regarding this complex topic.

After administering an antipyretic medication. Which intervention should the nurse implement?

Encouraging liberal fluid intake

A client with angina pectoris is being discharge from the hospital. What instruction should the nurse plan to include in this discharge teaching?

Engage in physical exercise immediately after eating to help decrease cholesterol levels. Walk briskly in cold weather to increase cardiac output Keep nitroglycerin in a light-colored plastic bottle and readily available. Avoid all isometric exercises, but walk regularly. Rationale: Isometric exercise can raise blood pressure for the duration of the exercise, which may be dangerous for a client with cardiovascular disease, while walking provides aerobic conditioning that improves ling, blood vessel, and muscle function. Client with angina should refrain from physical exercise for 2 hours after meals, but exercising does not decrease cholesterol levels. Cold water cause vasoconstriction that may cause chest pain. Nitroglycerin should be readily available and stored in a dark-colored glass bottle not C, to ensure freshness of the medication.

An elderly female is admitted because of a change in her level of sensorium. During the evening shift, the client attempts to get out bed and falls, breaking her left hip. Buck's skin traction is applied to the left leg while waiting for surgery. Which intervention is most important for the nurse to include in this client's plan care?

Ensure proper alignment of the leg in traction.

In assessing a client twelve hours following transurethral resection of the prostate (TURP), the nurse observes that the urinary drainage tubing contains a large amount of clear pale pink urine and the continuous bladder irrigation is infusing slowly. What action should the nurse implement?

Ensure that no dependent loops are present in the tubing.

A nurse is planning to teach infant care and preventive measures for sudden infant death syndrome (SIDS) to a group of new parents. What information is most important for the nurse to include?

Ensure that the infant's crib mattress is firm.

Before leaving the room of a confused client, the nurse notes that a half bow knot was used to attach the client's wrist restraints to the movable portion of the client's bed frame. What action should the nurse take before leaving the room?

Ensure that the knot can be quickly released. Tie the knot with a double turn or square knot. Move the ties so the restraints are secured to the side rails. Ensure that the restraints are snug against the client's wrist.

A vacuum-assistive closure (VAC) device is being use to provide wound care for a client who has stage III pressure ulcer on a below-the- knee (BKA) residual limb. Which intervention should the nurse implement to ensure maximum effectiveness of the device?

Ensure the transparent dressing has no tears that might create vacuum leaks

After receiving the first dose of penicillin, the client begins wheezing and has trouble breathing. The nurse notifies the healthcare provider immediately and received several prescriptions. Which medication prescription should the nurse administer first?

Epinephrine Injection, USP IV

A client admitted to the psychiatric unit diagnosed with major depression wants to sleep during the day, refuses to take a bath, and refuses to eat. Which nursing intervention should the nurse implement first?

Establish a structured routine for the client to follow.

A community health nurse is concerned about the spread of communicable diseases among migrant farm workers in a rural community. What action should the nurse take to promote the success of a healthcare program designed to address this problem?

Establish trust with community leaders and respect cultural and family values

Two clients ring their call bells simultaneously requesting pain medication. What action should the nurse implement first?

Evaluate both client's pain using a standardized pain scale

In monitoring tissue perfusion in a client following an above the knee amputation (aka), which action should the nurse include in the plan of care?

Evaluate closet proximal pulse. Asses skin elasticity of the stump. Observe for swelling around the stump. Note amount color of wound drainage. Rationale: A primary focus of care for a client with an AKA is monitoring for signs of adequate tissue perfusion, which include evaluating skin color and ongoing assessment of pulse strength.

A 12 year old client who had an appendectomy two days ago is receiving 0.9% normal saline at 50 ml/hour. The client's urine specific gravity is 1.035. What action should the nurse implement?

Evaluate postural blood pressure measurements Obtain specimen for uranalysis Encourage popsicles and fluids of choice Assess bowel sounds in all quadrants Rationale: specific gravity of urine is a measurement of hydration status (normal range of 1.010 to 1.025) which is indicative of fluid volume deficit when Sp Gr increases as urine becomes more concentrated.

A client who is admitted to the intensive care unit with syndrome of inappropriate antidiuretic hormone (SIADH) has developed osmotic demyelination. Which intervention should the nurse implement first?

Evaluate swallow. Rational: Osmotic demyelination, also known as central pontine myelinolysis, is nerve damage caused by the destruction of the myelin sheath covering nerve cells in the brainstem. The most common cause is a rapid, drastic change in sodium levels when a client is being treated for hyponatremia, a common occurrence in SIADH. Difficulty swallowing due to brainstem nerve damage should be care, but determining the client's risk for aspiration is most important.

The nurse ends the assessment of a client by performing a mental status exam. Which statement correctly describes the purpose of the mental status exam?

Evaluate the client's mood, cognition and orientation. Rational: the mental status exam assesses the client for abnormalities in cognitive functioning; potential thought processes, mood and reasoning, the other options listed are all components of the client's psychosocial assessment.

A young adult woman visits the clinic and learns that she is positive for BRCA1 gene mutation and asks the nurse what to expect next. How should the nurse respond?

Explain that counseling will be provided to give her information about her cancer risk. Rational: BRACA1or BRACA2 genetic mutation indicates an increased risk for developing breast or ovarian cancer and genetic counseling should be provided to explain the increased risk (A)to the client along with options for increased screening or preventative measures. (B) Is completed by the genetic counselor before the client undergoes genetic testing. a positive BRACA1test is not an indicator of the presence of cancer and (C and D) are not appropriate responses prior to genetic counseling.

A client with Alzheimer's disease (AD) is receiving trazodone (Desyrel), a recently prescribed atypical antidepressant. The caregiver tells the home health nurse that the client's mood and sleep patterns are improved, but there is no change in cognitive ability. How should the nurse respond to this information?

Explain that it may take several weeks for the medication to be effective Confirm the desired effect of the medication has been achieved. Notify the health care provider than a change may be needed. Evaluate when and how the medication is being administered to the client. Rationale: Trazodone o Desyrel, an atypical antidepressant, is prescribed for client with AD to improve mood and sleep.

When performing postural drainage on a client with Chronic Obstructive Pulmonary Disease (COPD), which approach should the nurse use?

Explain that the client may be placed in five positions

An adult male was diagnosed with stage IV lung cancer three weeks ago. His wife approaches the nurse and asks how she will know that her husband's death is imminent because their two adult children want to be there when he dies. What is the best response by the nurse?

Explain that the client will start to lose consciousness and his body system will slow down. Rationale: Expected signs of approaching death include noticeable changes in the client's level of consciousness and a slowing down of body systems. The nurse should answer the spouse's questions about the signs of imminent death rather than offering reassurance that may or may not be true. Other options listed may be implemented but the nurse should first answer the spouse's question directly.

An adult male is admitted to the emergency department after falling from a ladder. While waiting to have a computed tomography (CT) scan, he requests something for a severe headache. When the nurse offers him a prescribed does of acetaminophen, he asks for something stronger. Which intervention should the nurse implement?

Explain the reason for using only non-narcotics.

A clinical trial is recommended for a client with metastatic breast cancer, but she refuses to participate and tells her family that she does not wish to have further treatments. The client's son and daughter ask the nurse to try and convince their mother to reconsider this decision. How should the nurse respond?

Explore the client's decision to refuse treatment and offer support. Rationale: as long as the client is alert, oriented and aware of the disease prognosis, the healthcare team must abide by her decisions. Exploring the decision with the client and offering support provides a therapeutic interaction and allows the client to express her fears and concerns about her quality of life. Other options are essentially arguing with the client's decisions regarding her end of life treatment or diminish the opportunity for the client to discuss her feelings.

The nurse is planning care for a client who admits having suicidal thoughts. Which client behavior indicates the highest risk for the client acting on these suicidal thoughts?

Express feelings of sadness and loneliness Neglects personal hygiene and has no appetite Lacks interest in the activity of the family and friends Begin to show signs of improvement in affect Rationale: when a depressed client begins to show signs of improvement, it can be because the client has "figured out" how to be successful in committing suicide. Depressed clients, particularly those who have shown signs of potentially becoming suicidal, should be watched with care for an impending suicide attempt might be greater when the client appear suddenly happy, begin to give away possessions, or becomes more relaxed and talkative.

Diagnostic studies indicate that the elderly client has decreased bone density. In providing client teaching, which area of instruction is most important for the nurse to include?

Fall prevention measures.

The nurse is evaluating a client's symptoms, and formulates the nursing diagnosis, "high risk for injury due to possible urinary tract infection." Which symptoms indicate the need for this diagnosis?

Fever and dysuria.

A client with type 2 diabetes mellitus is admitted for frequent hyperglycemic episodes and a glycosylated hemoglobin (HbA1c) of 10%. Insulin glargine 10 units subcutaneously once a day at bedtime and a sliding scale with insulin aspart q6h are prescribed. What action should the nurse include in this client's plan of care?

Fingerstick glucose assessment q6h with meals Mix bedtime dose of insulin glargine with insulin aspart sliding scale dose Review with the client proper foot care and prevention of injury Do not contaminate the insulin aspart so that it is available for iv use Coordinate carbohydrate controlled meals at consistent times and intervals Teach subcutaneous injection technique, site rotation and insulin management

When conducting diet teaching for a client who was diagnosed with nutritional anemia in pregnancy, which foods should the nurse encourage the client to eat? (Select all that apply)

Fortified whole wheat cereals, whole-grain pasta, brown rice. Spinach, kale, dried raisins and apricots Rationale: Nutritional anemia in pregnancy should be supplemented with additional iron in the diet. Foods that are high in iron content are often protein based, whole grains (D), green leafy vegetables and dried fruits (E). (A, B, and C) are not iron rich sources.

The nurse discovers that an elderly client with no history of cardiac or renal disease has an elevated serum magnesium level. To further investigate the cause of this electrolyte imbalance, what information is most important for the nurse to obtain from the client's medical history?

Frequency of laxative use for chronic constipation.

A client with C-6 spinal cord injury rehabilitation. In the middle of the night the client reports a severe, pounding headache, and has observable piloerection or "goosebumps". The nurse should asses for which trigger?

Full bladder. Rational: a pounding headache is a sign of autonomic hyperreflexia, an acute emergency that occurs because of an exaggerated sympathetic response in a client with a high level spinal cord injury. Any stimulus below the level of injury can trigger autonomic hyperreflexia, but the most common cause is an overly distended bladder. The other options are unlikely to produce the manifestation of autonomic hyperreflexia.

A neonate with a congenital heart defect (CHD) is demonstrating symptoms of heart failure (HF). Which interventions should the nurse include in the infant's plan of care?

Give O2 at 6 L/nasal cannula for 3 repeated oximetry screens below 90% Administer diuretics via secondary infusion in the morning only Evaluate heart rate for effectiveness of cardio tonic medications Use high energy formula 30 calories/ounce at Q3 hours feeding via soft nipples Ensure Interrupted and frequent rest periods between procedures. Rationale: Pulse oximetry screening supports prescribed level of O2. HR provides an evaluative criterion for cardiac medications, which reduce heart rate, increase strength contractions (inotropic effects) and consequently affect systemic circulation and tissue oxygenation. Breast milk or basic formula provide 20 calories/ounce, so frequent feedings with high energy formula. D minimize fatigue is necessary.

The nurse is planning preoperative teaching plan of a 12-years old child who is scheduled for surgery. To help reduce the child anxiety, which action is the best for the nurse to implement?

Give the child syringes or hospital mask to play it at home prior to hospitalization. Include the child in pay therapy with children who are hospitalized for similar surgery. Provide a family tour of the preoperative unit one week before the surgery is scheduled. Provide doll an equipment to re-enact feeling associated with painful procedures. Rationale: School age children gain satisfaction from exploring and manipulating their environment, thinking about objectives, situations and events, and making judgments based on what they reason. A tour of the unit allows the child to see the hospital environment and reinforce explanation and conceptual thinking.

A female client presents in the Emergency Department and tells the nurse that she was raped last night. Which question is most important for the nurse to ask?

Has she taken a bath since the raped occurred? Rational: The priority action is collected the forensic evidence, so asking if the has taken a bath since the rape occurred is the most important information to obtain. Other options are used by law enforcement to determine the perpetrator and are not vital in providing client care at this time.

A client with rheumatoid arthritis (RA) starts a new prescription of etanercept (Enbrel) subcutaneously once weekly. The nurse should emphasize the importance of reporting problem to the healthcare provider?

Headache Joint stiffness Persistent fever Increase hunger and thirst Rationale: Enbrel decrease immune and inflammatory responses, increasing the client's risk of serious infection, so the client should be instructed to report a persistent fever, or other signs of infection to the healthcare provider.

The nurse receives a newborn within the first minutes after a vaginal delivery and intervenes to establish adequate respirations. What priority issue should the nurse address to ensure the newborn's survival?

Heat loss Hypoglycemia Fluid balance Bleeding tendencies

In assessing a client at 34-weeks' gestation, the nurse notes that she has a slightly elevated total T4 with a slightly enlarged thyroid, a hematocrit of 28%, a heart rate of 92 beats per minute, and a systolic murmur. Which finding requires follow-up?

Hematocrit of 28%. Rational: although physiologic anemia is expected in pregnancy, a hematocrit of 28% is below pregnant norms and could signify iron-deficiency anemia. Other options are normal finding pregnancy

The healthcare provider prescribes acarbose (Precose), an alpha-glucosidase inhibitor, for a client with Type 2 diabetes mellitus. Which information provides the best indicator of the drug's effectiveness?

Hemoglobin A1C (HbA1C) reading less than 7%

A client with arthritis has been receiving treatment with naproxen and now reports ongoing stomach pain, increasing weakness, and fatigue. Which laboratory test should the nurse monitor?

Hemoglobin. Rational: naproxen can cause gastric bleeding, so the nurse should monitor the client's hemoglobin to assess for possible bleeding. Other options are not likely to be affected by the used of naproxen and are not related to the client's current symptoms.

A 60-year-old female client asks the nurse about hormones replacement therapy (HRT) as a means preventing osteoporosis. Which factor in the client's history is a possible contraindication for the use of HRT?

Her mother and sister have a history of breast cancer

The nurse is auscultating a client's lung sounds. Which description should the nurse use to document this sound?

High pitched or fine crackles. Rhonchi High pitched wheeze Stridor

When providing diet teaching for a client with cholecystitis, which types of food choices the nurse recommend to the client?

High protein Low fat Low sodium High carbohydrate. Rationale: A client with cholecystitis is at risk of gall stones that can be move into the biliary tract and cause pain or obstruction. Reducing dietary fat decrease stimulation of the gall bladder, so bile can be expelled, along with possible stones, into the biliary tract and small intestine.

After six days on a mechanical ventilator, a male client is extubated and place on 40% oxygen via face mask. He is awake and cooperative, but complaining of a severe sore throat. While sipping water to swallow a medication, the client begins coughing, as if strangled. What intervention is most important for the nurse to implement?

Hold oral intake until swallow evaluation is done.

The nurse is teaching a mother of a newborn with a cleft lip how to bottle feed her baby using medela haberman feeder, which has a valve to control the release of milk and a slit nipple opening. The nurse discusses placing the nipple's elongated tip in the back of the oral cavity. What instructions should the nurse provide the mother about feedings?

Hold the newborn in an upright position. Rationale: the mother should be instructed to hold the infant during feedings in a sitting or upright position to prevent aspiration. Impaired sucking is compensated by the use of special feeding appliances and nipples such as the haberman feeder that prevents aspiration by adjusting the flow of mild according to the effort of the neonate. Squeezing the nipple base may introduce a volume that is greater than the neonate can coordinate swallowing. The preferred positon of an infant after feeding is on the right side to facilitate stomach emptying. Sucking difficulty impedes the neonate's intake of adequate nutrient needed for weight gain and water should be provided after the feeding to cleanse the oral cavity and not fill up the neonate's stomach

A group of nurse-managers is asked to engage in a needs assessment for a piece of equipment that will be expensed to the organization's budget. Which question is most important to consider when analyzing the cost-benefit for this piece of equipment?

How many departments can use this equipment? Will the equipment require annual repair? Is the cost of the equipment reasonable? Can the equipment be updated each year?

An adult client is exhibit the maniac stage of bipolar disorder is admitted to the psychiatric unit. The client has lost 10 pounds in the last two weeks and has no bathed in a week "I'm trying to start a new business and "I'm too busy to eat". The client is oriented to time, place, person but not situation. Which nursing problem has the greatest priority?

Hygiene-self-care deficit Imbalance nutrition Disturbed sleep pattern Self-neglect Rationale: The client's nutritional status has the highest priority at this time, and finger foods are often provided, so the client who is on the maniac phase of bipolar disease can receive adequate nutrition. Other options are nursing problems that should also be addresses with the client's plan of care, but at this stage in the client's treatment, adequate nutrition is a priority

The nurse walks into a client's room and notices bright red blood on the sheets and on the floor by the IV pole. Which action should the nurse take first?

Identify the source and amount of bleeding. Rationale: the nursed should first assess the client to determine the action that should be taken. Patient safety is the priority; other options are not priority.

A client presents in the emergency room with right-sided facial asymmetry. The nurse asks the client to perform a series of movements that require use of the facial muscles. What symptoms suggest that the client has most likely experience a Bell's palsy rather than a stroke?

Inability to close the affected eye, raise brow, or smile

Assessment by the home health nurse of an older client who lives alone indicates that client has chronic constipation. Daily medications include furosemide for hypertension and heart failure and laxatives. To manage the client's constipation, which suggestions should the nurse provide? (Select all that apply)

Include oatmeal with stewed pruned for breakfast as often as possible. Increase fluid intake by keeping water glass next to recliner. Recommend seeking help with regular shopping and meal preparation. Rational: older adult are at higher risk for chronic constipation due to decreased gastrointestinal muscle tone leading to reduce motility. Oatmeal with prunes increases dietary fiber and bowel stimulation, thereby decreasing need for laxatives. Increased fluid intake also decreases constipations. Assistance with food preparation might help the client eat more fresh fruits and vegetables and result on less reliance on microwaved and fast foods, which are usually high in sodium and fat with little fiber. Laxatives can be reduced gradually by improving the diet, without resorting to using enemas.

The nurse is developing the plan of care for a client with pneumonia and includes the nursing diagnosis of "Ineffective airway clearance related to thick pulmonary secretions." Which intervention is most important for the nurse to include in the client's plan of care?

Increase fluid intake to 3,000 ml/daily

A client who sustained a head injury following an automobile collision is admitted to the hospital. The nurse include the client's risk for developing increased intracranial pressure (ICP) in the plan of care. Which signs indicate to the nurse that ICP has increased?

Increased Glasgow coma scale score. Nuchal rigidity and papilledema. Confusion and papilledema Periorbital ecchymosis. Rationale: papilledema is always an indicator of increased ICP, and confusion is usually the first sign of increased ICP. Other options do not necessarily reflect increased ICP.

A young adult client is admitted to the emergency room following a motor vehicle collision. The client's head hit the dashboard. Admission assessment include: Blood pressure 85/45 mm Hg, temperature 98.6 F, pulse 124 beat/minute and respirations 22 breath/minute. Based on these data, the nurse formulates the first portion of nursing diagnosis as " Risk of injury" What term best expresses the "related to" portion of nursing diagnosis?

Infection Increase intracranial pressure Shock Head Injury.

The psychiatric nurse is talking to a newly admitted client when a male client diagnosed with antisocial behavior intrudes on the conversation and tells the nurse, "I have to talk to you right now! It is very important!" how should the nurse respond to this client?

Inform him that the nurse is busy admitting a new client and will talk to him later. Rational: the psychiatric nurse must set limits with antisocial behavior so that appropriate behavior is demonstrated. Interrupting a conversation is rude and inappropriate, so telling the client that they can talk later is the best course of action. Other options may cause the client to become angry and they do not address the client's behavior. The nurse should not involve this client with newly admitted client's admission procedure.

What action should the school nurse implement to provide secondary prevention to a school-age children?

Initiate a hearing and vision screening program for first-graders. Rationale: Community care occurs at primary, secondary, and tertiary levels of prevention. Primary prevention involves interventions to reduce the incidence of disease. Secondary prevention includes screening programs to detect disease. Tertiary prevention provides treatment directed toward clinically apparent disease. Secondary prevention focuses on screaming children for a specific disease processes such as hearing and vision screening. The other options are not examples of secondary prevention.

When implementing a disaster intervention plan, which intervention should the nurse implement first?

Initiate the discharge of stable clients from hospital units Identify a command center where activities are coordinated Assess community safety needs impacted by the disaster Instruct all essential off-duty personnel to report to the facility

What is the priority nursing action when initiating morphine therapy via an intravenous patient-controlled analgesia (PCA) pump?

Initiate the dosage lockout mechanism on the PCA pump Instruct the client to use the medication before the pain becomes severe Assess the abdomen for bowel sounds. Assess the client ability to use a numeric pain scale

The nurse is preparing a client who had a below-the-knee (BKA) amputation for discharge to home. Which recommendations should the nurse provide this client? (Select all that apply)

Inspect skin for redness. Use a residual limb shrinker. Wash the stump with soap and water. Rationale: Several actions are recommended for home care following an amputation. The skin should be inspected regularly for abnormalities such as redness, blistering, or abrasions. A residual limb shrinker should be applied over the stump to protect it and reduce edema. The stump should be washed daily with a mild soap and carefully rinse and dried. The client should avoid cleansing with alcohol because it can dry and crack the skin. Range of motion should be done daily.

A child newly diagnosed with sickle cell anemia (SCA) is being discharged from the hospital. Which information is most important for the nurse to provide the parents prior to discharge?

Instructions about how much fluid the child should drink daily. Rationale: It is essential that the child and family understands the importance of adequate hydration in preventing the stasis-thrombosis-ischemia cycle of a crisis that has a specific plan for hydration is developed so that a crisis can be delayed. Other choices listed are not the most important topics to include in the discharge teaching.

The nurse is interacting with a female client who is diagnosed with postpartum depression. Which finding should the nurse document as an objective signs of depression? (Select all that apply)

Interacts with a flat affect. Avoids eye contact. Has a disheveled appearance.

The nurse is caring for a client who is taking a macrolide to treat a bacterial infection. Which finding should the nurse report to the healthcare provider before administering the next dose?

Jaundice Nausea Fever Fatigue

A client with a history of cirrhosis and alcoholism is admitted with severe dyspnea and ascites. Which assessment finding warrants immediate intervention by the nurse?

Jaundice skin tone Muffled heart sounds Pitting peripheral edema Bilateral scleral edema Rationale: Muffled heart sounds may indicative fluid build-up in the pericardium and is life- threatening. The other one are signs of end stage liver disease related to alcoholism but are not immediately life- threatening.

The nurse is teaching a client how to perform colostomy irrigations. When observing the client's return demonstration, which action indicated that the client understood the teaching?

Keeps the irrigating container less than 18 inches above the stoma

The nurse caring for a client with acute renal fluid (ARF) has noted that the client has voided 800 ml of urine in 4 hours. Based on this assessment, what should the nurse anticipate that client will need?

Large amounts of fluid and electrolyte replacement.

During the admission assessment, the nurse auscultates heart sounds for a client with no history of cardiovascular disease. Where should the nurse listen when assessing the client's point of maximal impulse (PMI) (Click the chosen location. To change, click on a new location)

Left Fourth Intercostal Space

An adult male client is admitted to the emergency room following an automobile collision in which he sustained a head injury. What assessment data would provide the earliest that the client is experiencing increased intracranial pressure (ICP)?

Lethargy Decorticate posturing Fixed dilated pupil Clear drainage from the ear. Rationale: Lethargy is the earliest sign of ICP along with slowing of speech and response to verbal commands. The most important indicator of increase ICP is the client's level or responsiveness or consciousness. B and C are very late signs of ICP.

A client is receiving lactulose (Portalac) for signs of hepatic encephalopathy. To evaluate the client's therapeutic response to this medication, which assessment should the nurse obtain?

Level of consciousness. Rationale: Colonic bacteria digest lactulose to create a drug-induces acidic and hyperosmotic environment that draws water and blood ammonia into the colon and coverts ammonia to ammonium, which is trapped in the intestines and cannot be reabsorbed into the systemic circulation. This therapeutic action of lactulose is to reduce serum ammonia levels, which improves the client's level of consciousness and metal status.

The nurse is preparing a discharge teaching plan for a client who had a liver transplant. Which instruction is most important to include in this plan?

Limit intake fatty foods for one month after surgery. Notify the healthcare provider if edema occurs. Increase activity and exercise gradually, as tolerated. Avoid crowds for first two months after surgery. Rationale: Cyclosporine immunosuppression therapy is vital in the success of liver transplantation and can increase the risk for infection, which is critical in the first two months after surgery. Fever is often.

An older adult male is admitted with complications related to chronic obstructive pulmonary disease (COPD). He reports progressive dyspnea that worsens on exertion and his weakness has increased over the past month. The nurse notes that he has dependent edema in both lower legs. Based on these assessment findings, which dietary instruction should the nurse provide?

Limit the intake of high calorie foods. Eat meals at the same time daily. Maintain a low protein diet. Restrict daily fluid intake. Rationale: the client is exhibiting signs of cor pulmonale, a complication of COPD that causes the right side of the heart to fail. Restricting fluid intake to 1000 to 2000 ml/day, eating a high-calorie diet at small frequent meals with foods that are high in protein and low in sodium can help relive the edema and decrease workload on the right-side of the heart.

A client is admitted with metastatic carcinoma of the liver, ascites, and bilateral 4+ pitting edema of both lower extremities. When the client complains that the antiembolic stocking are too constricting, which intervention should the nurse implement?

Maintain both lower extremities elevated on pillows.

When assessing a multigravida the first postpartum day, the nurse finds a moderate amount of lochia rubra, with the uterus firm, and three fingerbreadths above the umbilicus. What action should the nurse implement first?

Massage the uterus to decrease atony Check for a destined bladder Increase intravenous infusion Review the hemoglobin to determined hemorrhage Rationale: a fundus that is dextroverted (up to the right) and elevated above the umbilicus is indicative of bladder distension/urine retention.

A client at 30 week gestation is admitted due to preterm labor. A prescription of terbutaline sulfate 8.35 mg is gives subcutaneously. Based on which finding should the nurse withhold the next dose of this drug?

Maternal pulse rate of 162 beats per min

A client with type 2 diabetes mellitus is admitted for antibiotic treatment for a leg ulcer. To monitor the client for the onset of hyperosmolar hyperglycemic nonketotic syndrome (HHNS), what actions should the nurse take? (Select all that apply)

Measure blood glucose. Monitor vital signs. Assessed level of consciousness. Rationale: Blood glucose greater than 600 mg/dl (33.3 mmol/L SI), vital sign changes in mental awareness are indicators of possible HHNS. Urine ketones are monitored in diabetic ketoacidosis. Wound culture is performed prior to treating the wound infection but is not useful in monitoring for HHNS.

While changing a client's chest tube dressing, the nurse notes a crackling sensation when gentle pressure is applied to the skin at the insertion site. What is the best action for the nurse to take?

Measure the area of swelling and crackling. Rational: a crackling sensation, or crepitus, indicates subcutaneous emphysema, or air leaking into the skin. This area should be measured and the finding documented. Other options are not indicated for crepitus.

While assisting a client who recently had a hip replacement into a bed pan, the nurse notices that there is a small amount of bloody drainage on the surgical dressing, the client's skin is warm to the touch, and there is a strong odor from the urine. Which action should the nurse take?

Measure the client's oral temperature. Rationale: The strong odor from the urine and skin that is warm to the touch may indicate that the client has a urinary tract infection. Assessing the client's temperature provides objective information regarding infection that can be reported to the healthcare provider. Urine should be obtained via a clean catch, not the bed pan where it has been contaminated. The drainage on the dressing is normal and does not require direct conservation at this time. An indwelling catheter should be avoided if possible because it increases the risk of infection.

Ten years after a female client was diagnosed with multiple sclerosis (MS), she is admitted to a community palliative care unit. Which intervention is most important for the nurse to include in the client's plan of care?

Medicate as needed for pain and anxiety.

Which intervention should the nurse include in the plan of care for a child with tetanus?

Minimize the amount of stimuli in the room

A client with cirrhosis of the liver is admitted with complications related to end stage liver disease. Which intervention should the nurse implement? (Select all that apply.)

Monitor abdominal girth. Increase oral fluid intake to 1500 ml daily. Report serum albumin and globulin levels. Provide diet low in phosphorous. Note signs of swelling and edema. Rational: monitoring for increasing abdominal girth and generalized tissue edema and swelling are focused assessments that provide data about the progression of disease related complications. In advanced cirrhosis, liver function failure results in low serum albumin and serum protein levels, which caused third spacing that results in generalized fluid retention and ascites. Other options are not indicated in end stage liver disease.

A client with acute renal failure (ARF) is admitted for uncontrolled type 1 diabetes Mellitus and hyperkalemia. The nurse administers an IV dose of regular insulin per sliding scale. Which intervention is the most important for the nurse to include in this client's plan of care?

Monitor the client's cardiac activity via telemetry. Rational: as insulin lowers the blood glucose of a client with diabetic ketoacidosis (DKA), potassium returns to the cell but may not impact hyperkalemia related to acute renal failure. The priority is to monitor the client for cardiac dysrhythmias related to abnormal serum potassium levels. IV access, assessment of glucose level, and monitoring urine output are important interventions, but do not have the priority of monitoring cardiac function.

While monitoring a client during a seizure, which interventions should the nurse implement? (Select all that apply)

Move obstacle away from client Monitor physical movements Observe for a patent airway Record the duration of the seizure

A new member joins the nursing team spreads books on the table, puts items on two chairs, and sits on a third chair. The members of the group are forced to move closer and remove their possessions from the table what action should the nurse leader take?

Move to welcome and accommodate a new person Ask the new person to move belonging to accommodate others Tell the new person to move belongings because of limited space Bring in additional chairs so that all staff members can be seated

An older client is admitted to the intensive care unit with severe abdominal pain, abdominal distention, and absent bowel sound. The client has a history of smoking 2 packs of cigarettes daily for 50 years and is currently restless and confused. Vital signs are: temperature 96`F, heart rate 122 beats/minute, respiratory rate 36 breaths/minute, mean arterial pressure(MAP) 64 mmHg and central venous pressure (CVP) 7 mmHg. Serum laboratory findings include: hemoglobin 6.5 grams/dl, platelets 6o, 000, and white blood cell count (WBC) 3,000/mm3. Based on these findings this client is at greatest risk for which pathophysiological condition?

Multiple organ dysfunction syndrome (MODS) Disseminated intravascular coagulation (DIC) Chronic obstructive disease. Acquired immunodeficiency syndrome (AIDS) Rational: MODS are a progressive dysfunction of two or more major organs that requires medical intervention to maintain homeostasis. This client has evidence of several organ systems that require intervention, such as blood pressure, hemoglobin, WBC, and respiratory rate. DIC may develop as a result of MODS. The other options are not correct.

The nurse is auscultating is auscultating a client's heart sounds. Which description should the nurse use to document this sound? (Please listen to the audio file to select the option that applies.)

Murmur s1 s2 pericardial friction rub s1 s2 s3

The nurse is preparing a teaching plan for an older female client diagnosed with osteoporosis. What expected outcome has the highest priority for this client?

Names 3 home safety hazards to be resolve immediately. Rational: a major teaching goal for an elderly client with osteoporosis is maintenance of safety to prevent falls. Injury due to a fall, usually resulting in a hip fracture, can result in reduced mobility and associated complications. Other goals are also important when teaching clients who have osteoporosis, but they do not have the priority of preventing falls, which relates to safety

A male client who was diagnosed with viral hepatitis A 4 weeks ago returns to the clinic complaining of weakness and fatigue. Which finding is most important for the nurse to report to the healthcare provider?

New onset of purple skin lesions.

After receiving report, the nurse can most safely plan to assess which client last? The client with...

No postoperative drainage in the Jackson-Pratt drain with the bulb compressed. Rationale: The most stable client is the one with a functioning drainage device and no drainage. This client can most safely be assesses last. Other clients are either actively bleeding, have an obstruction in the nasogastric tube which may result in vomiting, or may be bleeding and / or may have a malfunction in the Hemovac® drain.

A client is admitted to the emergency department with a respiratory rate of 34 breaths per minute and high pitched wheezing on inspiration and expiration, the medical diagnosis is severe exacerbation of asthma. Which assessment finding, obtained 10 min after the admission assessment, should the nurse report immediately to the emergency department healthcare provider?

No wheezing upon auscultation of the chest.

Two days after admission a male client remembers that he is allergic to eggs, and informs the nurse of the allergy. Which actions should the nurse implement? (Select all that apply)

Notify the food services department of the allergy. Enter the allergy information in the client's record. Add egg allergy to the client's allergy arm band.

A mother brings her 3-year-old son to the emergency room and tells the nurse the he has had an upper respiratory infection for the past two days. Assessment of the child reveals a rectal temperature of 102 F. he is drooling and becoming increasingly more restless. What action should the nurse take first?

Notify the healthcare provider and obtain a tracheostomy tray

An older male adult resident of long-term care facility is hospitalized for a cardiac catheterization that occurred yesterday. Since the procedure was conducted, the client has become increasingly disoriented. The night shift nurse reports that he attempted to remove the sandbag from his femoral artery multiple times during the night. What actions should the nurse take? (Select all that apply.)

Notify the healthcare provider of the client's change in mental status. Include q2 hour's reorientation in the client's plan of care.

After the risk and benefits of having a cardiac catheterization are reviewed by the healthcare provider, an older adult with unstable angina is scheduled for the procedure. When the nurse presents the consent form for signature, the client asks how the wires will keep a heart heating during the procedure. What action should the nurse take?

Notify the healthcare provider of the client's lack of understanding. Rational: the nurse is only witnessing the signature, and is not responsible for the client's understanding of the procedure. The healthcare provider needs to clarify any questions and misconceptions. Explaining the procedure again is the healthcare provider's legal responsibility. The other options are not indicated.

Four hours after surgery, a client reports nausea and begins to vomit. The nurse notes that the client has a scopolamine transdermal patch applied behind the ear. What action should the nurse take?

Notify the healthcare provider of the vomiting. Rational: transdermal scopolamine is used to prevent nausea and vomiting from anesthesia and surgery. The nurse should notify the healthcare provider if the medication is ineffective. The patch should be applied behind the ear and should remain in place to reduce the nausea and vomiting. Nausea and vomiting are no side effects of the medication.

A client with leukemia undergoes a bone marrow biopsy. The client's laboratory values indicate the client has thrombocytopenia. Based on this data, which nursing assessment is most important following the procedure?

Observe aspiration site. Assess body temperature Monitor skin elasticity Measure urinary output

The nurse is teaching a male adolescent recently diagnosed with type 1diabetes mellitus (DM) about self-injecting insulin. Which approach is best for the nurse to use to evaluate do you effectiveness of the teaching?

Observe him as he demonstrates self-injection technique in another diabetic adolescent. Rational: watching the adolescent perform the procedure with another adolescent provides peer support the most information regarding his skill with self-injection. Other options do not provide information about the effectiveness of nurse's teaching.

An older adult client with heart failure (HF) develops cardiac tamponade. The client has muffled, distant, heart sounds, and is anxious and restless. After initiating oxygen therapy and IV hydration, which intervention is most important for the nurse to implement?

Observe neck for jugular vein distention Notify healthcare provider to prepare for pericardiocentesis Asses for paradoxical blood pressure Monitor oxygen saturation (Sp02) via continuous pulse oximetry Rationale: Cardiac tamponade is pressure on the heart that occurs when blood or fluid builds up in the space between the heart muscle (myocardium) and the outer covering sac of the heart (pericardium). In this condition, blood or fluid collects in the pericardium, the sac surrounding the heart. This prevents the heart ventricles from expanding fully. The excess pressure from the fluid prevents the heart from working properly. As a result, the body does not get enough blood.

A client with myasthenia Gravis (MG) is receiving immunosuppressive therapy. Review recent laboratory test results show that the client's serum magnesium level has decreased below the normal range. In addition to contacting the healthcare provider, what nursing action is most important?

Observe rhythm on telemetry monitor. Rationale: If not treated a low little Serum magnesium level can affect myocardial depolarization leading to a lethal arrhythmia, and the nurse should assess for dysrhythmias before contacting the healthcare provider. Other choices are common in MG but do not contribute the Safety risk of low magnesium levels.

An adult client with schizophrenia begin treatment three days ago with the Antipsychotic risperidone. The client also received prescription for trazodone as needed for sleep and clonazepam as needed for severe anxiety. When the client reports difficulty with swallowing, what action should the nurse take?

Obtain a prescription for an anticholinergic medication. Rationale: Antipsychotic medications have an extrapyramidal side effects one of which is difficult to swallowing the nurse should obtain a prescription for an anticholinergic medication which is used for the treatment of extrapyramidal symptoms. Other options are not warranted actions based on the symptoms presented

An older male client arrives at the clinic complaining that his bladder always feels full. He complains of weak urine flow, frequent dribbling after voiding, and increasing nocturia with difficulty initiating his urine stream. Which action should the nurse implement?

Obtain a urine specimen for culture and sensitivity Palpate the client's suprapubic area for distention Advise the client to maintain a voiding diary for one week Instruct in effective technique to cleanse the glans penis Rationale: the client is exhibiting classic signs of an enlarge prostate gland, which restricts urine flow and cause bothersome lower urinary tract symptoms (LUTS) and urinary retention, which is characterized by the client's voiding patterns and perception of incomplete bladder emptying.

A client with possible acute kidney injury (AKI) is admitted to the hospital and mannitol is prescribed as a fluid challenge. Prior to carrying out this prescription, what intervention should the nurse implement?

Obtain vital signs and breath sounds. Rational: the client's baseline cardiovascular status should be determined before conducting the fluid challenge. If the client manifests changes in the vital signs and breath sounds associated with pulmonary edema, the administration of the fluid challenge should be terminate. Other options would not assure a safe administration of the medication.

The nurse instructs an unlicensed assistive personnel (UAP) to turn an immobilized elderly client with an indwelling urinary catheter every two hours. What additional action should the nurse instruct the UAP to take each time the client is turned?

Offer the client oral fluids. Rationale: Increasing oral fluid intake reduces the risk of problems associated with immobility, so the UAP should be instructed to offer the client oral fluids every two hours, or whenever turning he client. It is not necessary to empty the urinary bag or feed the client every two hours. Assessment is a nursing function, and UAPs do not have the expertise to perform assessment of breath sounds.

A young adult female college student visits the health clinic in early winter to obtain birth control pills. The clinic nurse asks if the student has received an influenza vaccination. The student stated she did not receive vaccination because she has asthma. How should the nurse respond?

Offer to provide the influenza vaccination to the student while she is at the clinic. Rationale: person with asthma are at increased risk related to influenza and should receive the influenza vaccination prior to or during influenza season. Waiting until the start of the next season places the student at risk for the current season. The vaccination does not increase risk for persons with asthma but the nasal spray may result in increased wheezing after receiving that form of the vaccination

Following a motor vehicle collision, an adult female with a ruptured spleen and a blood pressure of 70/44, had an emergency splenectomy. Twelve hours after the surgery, her urine output is 25 ml/hour for the last two hours. What pathophysiological reason supports the nurse's decision to report this finding to the healthcare provider?

Oliguria signals tubular necrosis related to hypoperfusion

Based on principles of asepsis, the nurse should consider which circumstance to be sterile?

One inch- border around the edge of the sterile field set up in the operating room A wrapped unopened, sterile 4x4 gauze placed on a damp table top. An open sterile Foley catheter kit set up on a table at the nurse waist level Sterile syringe is placed on sterile area as the nurse riches over the sterile field. Rationale: A sterile package at or above the waist level is considered sterile. The edge of sterile field is contaminated which include a 1-inch border (A). A sterile objects become contaminated by capillary action when sterile objects become in contact with a wet contaminated surface.

A client has an intravenous fluid infusing in the right forearm. To determine the client's distal pulse ratemost accurately, which action should the nurse implement?

Palpate at the radial pulse site with the pads of two or three fingers.

A native-American male client diagnosed with pneumonia, states that in addition to his prescribed medical treatment of IV antibiotics he wishes to have a spiritual cleaning performed. Which outcome statement indicates that the best plan of care was followed?

Participated actively in all treatments regimens.\ Rationale: indicates active participation by the client, which is required for treatment to be successful. The best plan of care should incorporate the valued and treatments of both cultures and in this case there is no apparent cultural clash between the two forms of treatment. The client has already identify he's cultural values (A). (B) Only considers one of the two treatment modalities desired by the client the client has already chosen how he wishes to assimilate his cultural values with the prescribed medical treatment (D).

When conducting diet teaching for a client who is on a postoperative soft diet, which foods should eat? (Select all that apply)

Pasta, noodles, rice. Egg, tofu, ground meat. Mashed, potatoes, pudding, milk. Rational: a client's postoperative diet is commonly progressed as tolerated. A soft diet includes foods that are mechanically soft in texture (pasta, egg, ground meat, potatoes, and pudding. High fiber foods that require thorough chewing and gas forming foods, such as cruciferous vegetables and fresh fruits with skin, grains and seeds are omitted.

The mother of the 12- month-old with cystic fibrosis reports that her child is experiencing increasing congestion despite the use of chest physical therapy (CPT) twice a day, and has also experiences a loss of appetite. What instruction should the nurse provide?

Perform CPT after meals to increase appetite and improve food intake. CPT should be performed more frequently, but at least an hour before meals. Stop using CPT during the daytime until the child has regained an appetite. Perform CPT only in the morning, but increase frequency when appetite improves. Rationale: CPY with inhalation therapy should be performed several times a day to loosen the secretions and move them from the peripheral airway into the central airways where they can be expectorated. CPT should be done at least one hour before meals or two hours after meals.

The rapid response team's detects return of spontaneous circulation (ROSC) after 2 min of continuous chest compressions. The client has a weak, fast pulse and no respiratory effort, so the healthcare provider performs a successful oral, intubation. What action should the nurse implement?

Perform bilateral chest auscultation.

The nurse is demonstrating correct transfer procedures to the unlicensed assisted personnel (UAP) working on a rehabilitation unit. The UAPs ask the nurse how to safely move a physically disabled client from the wheelchair to a bed. What action should the nurse recommended?

Place a client's locked wheelchair on the client's strong side next to the bed. Rational: Placing the wheelchair on the client's strong side offers the greatest stability for the transfer. Holding the client arm's length or pulling from the opposite site of the bed reflect poor body mechanism. Using a gait belt offers additional safety for the client, but should be done after the wheelchair has be put into the proper place and the wheels have been locked and before the client has assumed a standing position.

At 40 week gestation, a laboring client who is lying is a supine position tells the nurse that she has finally found a comfortable position. What action should the nurse take?

Place a wedge under the client's right hip. Rationale: Hypotension from pressure on the vena cava is a risk for the full-term client. Placing a wedge under the right hip will relieve pressure on the vena cava. Other options will either not relieve pressure on the vena cava or would not allow the client the remaining her position of choice.

In caring for the body of a client who just died, which tasks can be delegate to the unlicensed assistive personnel (UAP)? (Select all that apply.)

Place personal religious artifacts on the body. Attach identifying name tags to the body. Follow cultural beliefs in preparing the body.

Oral antibiotics are prescribed for an 18-month-old toddler with severe otitis media. An antipyrine and benzocaine-otic also prescribed for pain and inflammation. What instruction should the nurse emphasize concerning the installation of the antipyrine/benzocaine otic solution?

Place the dropper on the upper outer ear canal and instill the medication slowly. Warm the medication in the microwave for 10 seconds before instilling. Keep the medication refrigerated between administrations. Have the child lie with the ear up for one to two minute after installation.

The nurse is caring for a one week old infant who has a ventriculoperitoneal (VP) shunt that was placed 2 days after birth. Which findings are an indication of a postoperative complication?

Poor feeding and vomiting Leakage of CSF from the incisional site Hyperactive bowel sound Abdominal distention WBC count of 10000/mm3

The nurse is teaching a postmenopausal client about osteoporosis prevention. The client reports that she smokes 2 packs of cigarettes a day and takes 750 mg calcium supplements daily. What information should the nurse include when teaching this client about osteoporosis prevention?

Postmenopausal women need an intake of at least 1,500 mg of calcium daily.

Following an esophagogastroduodenoscopy (EGD) a male client is drowsy and difficult to arouse, and his respiration are slow and shallow. Which action should the nurse implement? Select all that apply.

Prepare medication reversal agent Check oxygen saturation level Apply oxygen via nasal cannula Initiate bag- valve mask ventilation. Begin cardiopulmonary resuscitation Rationale: Sedation, given during the procedure may need to be reverse if the client does not easily wake up. Oxygen saturation level should be asses, and oxygen applied to support respiratory effort and oxygenation. The client is still breathing so the bag- valve mask ventilation and CPR are not necessary.

In preparing a diabetes education program, which goal should the nurse identify as the primary emphasis for a class on diabetes self-management?

Prepare the client to independently treat their disease process Reduce healthcare costs related to diabetic complications Enable clients to become active participating in controlling the disease process Increase client's knowledge of the diabetic disease process and treatment options. Rationale: The primary goal of diabetic self- management education is to enable the client to become an active participant in the care and control of disease process, matching levels of self- management to the abilities of the individual client. The goal is to place the client in a cooperative or collaborative role with healthcare professional rather than (A)

A client with coronary artery disease who is experiencing syncopal episodes is admitted for an electrophysiology study (EPS) and possible cardiac ablation therapy. Which intervention should the nurse delegate to the unlicensed assistive personnel (UAP)?

Prepare the skin for procedure. Identify client's pulse points Witness consent for procedure Check telemetry monitoring

When entering a client's room, the nurse discovers that the client is unresponsive and pulseless. The nurse initiate CPR and Calls for assistance. Which action should the nurse take next?

Prepare to administer atropine 0.4 mg IVP Gather emergency tracheostomy equipment Prepare to administer lidocaine at 100 mg IVP Place cardiac monitor leads on the client's chest. Rationale: Before further interventions can be done, the client's heart rhythm must be determined. This can be done by connecting the client to the monitor. A or C are not a first line drug given for any of the life threatening, pulses dysrhythmias

While assessing a radial artery catheter, the client complains of numbness and pain distal to the insertion site. What interventions should the nurse implement?

Promptly remove the arterial catheter from the radial artery.

The nurse is teaching a group of clients with rheumatoid arthritis about the need to modify daily activities. Which goal should the nurse emphasize?

Protect joint function Improve circulation Control tremors Increase weight bearing

A 56-years-old man shares with the nurse that he is having difficulty making decision about terminating life support for his wife. What is the best initial action by the nurse?

Provide an opportunity for him to clarify his values related to the decision Encourage him to share memories about his life with his wife and family Advise him to seek several opinions before making decision Offer to contact the hospital chaplain or social worker to offer support. Rationale: When a client is faced with a decisional conflict, the nurse should first provide opportunities for the client to clarify values important in the decision. The rest may also be beneficial once the client as clarified the values that are important to him in the decision-making process.

A client with a cervical spinal cord injury (SCI) has Crutchfield tongs and skeletal traction applied as a method of closed reduction. Which intervention is most important for the nurse to include in the client's a plan of care?

Provide daily care of tong insertion sites using saline and antibiotic ointment

A client with emphysema is being discharged from the hospital. The nurse enters the client's room to complete discharge teaching. The client reports feeling a little short of breath and is anxious about going home. What is the best course of action?

Provide only necessary information in short, simple explanations with written instructions to take home. Rationale: Simple, short explanations should be provided. Information is not retained when the recipient is anxious, and too much information can increase worry. Ethically, discharge instructions may not be postponed.

While assessing a client's chest tube (CT), the nurse discovers bubbling in the water seal chamber of the chest tube collection device. The client's vital signs are: blood pressure of 80/40 mmHg, heart rate 120 beats/minutes, respiratory rate 32 breaths/minutes, oxygen saturation 88%. Which interventions should the nurse implement?

Provide supplemental oxygen Auscultate bilateral lung fields Administer a nebulizer treatment Reinforce occlusive CT dressing Give PRN dose of pain medication Rationale: the air bubbles indicate an air leak from the lungs, the chest tube site, or the chest tube collection system. Providing oxygen improves the oxygen saturation until the leak has been resolved. Auscultating the lung fields helps to identify absent or decrease lung sound due to collapsing lung.

A man expresses concern to the nurse about the care his mother is receiving while hospitalized. He believes that her care is not based on any ethical standards and ask what type of care he should expect from a public hospital. What action should the nurse take?

Provide the man and his mother with a copy of the Patient's Bill of Rights

A low-risk primigravida at 28-weeks gestation arrives for her regular antepartal clinic visit. Which assessment finding should the nurse consider within normal limits for this client?

Pulse increase of 10 beats/minute.

While undergoing hemodialysis, a male client suddenly complains of dizziness. He is alert and oriented, but his skin is cool and clammy. His vital signs are: heart rate 128 beats/minute, respirations 18 breaths/minute, and blood pressure 90/60. Which intervention should the nurse implement first?

Raise the client's legs and feet

A male client with cirrhosis has ascites and reports feeling short of breath. The client is in semi Fowler position with his arms at his side. What action should the nurse implement?

Raise the head of the bed to a Fowler's position and support his arms with a pillow

Which assessment is more important for the nurse to include in the daily plan of care for a client with a burned extremity?

Range of Motion Distal pulse intensity Extremity sensation Presence of exudate Rationale: Distal pulse intensity assesses the blood flow through the extremity and is the most important assessment because it provides information about adequate circulation to the extremity. Range of motions evaluates the possibility of long term contractures sensation. C evaluates neurological involvement, and exudate. D provides information about wound infection, but this assessment do not have the priority of determining perfusion to the extremity.

A client is admitted with an epidural hematoma that resulted from a skateboarding accident. To differentiate the vascular source of the intracranial bleeding, which finding should the nurse monitor?

Rapid onset of decreased level of consciousness.

The nurse assesses a child in 90-90 traction. Where should did nurse assess for signs of compartment syndrome?

Rationale: compartment syndrome is the result of swelling and subsequent reduction in circulation to the area distal to the compartment. This can be a complication of traumatic injury and cast administration, so it is important to assess circulation distal to the casted prolonged capillary refill.

A client who had a right hip replacement 3 day ago is pale has diminished breath sound over the left lower lung fields, a temperature of 100.2 F, and an oxygen saturation rate of 90%. The client is scheduled to be transferred to a skilled nursing facility (SNF) tomorrow for rehabilitative critical pathway. Based on the client's symptoms, what recommendation should the nurse give the healthcare provider?

Reassess readiness for SNF transfer.

A client who is newly diagnosed with type 2 diabetes mellitus (DM) receives a prescription for metformin (Glucophage) 500 mg PO twice daily. What information should the nurse include in this client's teaching plan? (Select all that apply.)

Recognize signs and symptoms of hypoglycemia. Report persist polyuria to the healthcare provider. Take Glucophage with the morning and evening meal.

The nurse is preparing a community education program on osteoporosis. Which instruction is helpful in preventing bone loss and promoting bone formation?

Recommend weigh bearing physical activity

When assessing the surgical dressing of a client who had abdominal surgery the previous day, the nurse observes that a small amount of drainage is present on the dressing and the wound's Hemovac suction device is empty with the plug open. How should the nurse respond?

Recompress the wound suction device and secure to plug. Rationale: The plug of a wound suction device, such as a Hemovac, should be closed after compressing the device to apply gentle suction in a closed surgical wound to facilitate the evacuation of subcutaneous fluids into the device. Compressing the device and securing the plug should restore function of the closed wound device. A small amount of drainage should be marked on the dressing, but replacing the dressing is not necessary and the nurse should not remove the device. Other options are not indicated

A client with diabetic peripheral neuropathy has been taking pregabalin (Lyrica) for 4 days. Which finding indicates to the nurse that the medication is effective?

Reduced level of pain Full volume of pedal pulses Granulating tissue in foot ulcer Improved visual acuity

An 18-year-old female client is seen at the health department for treatment of condylomata acuminate (perineal warts) caused by the human papillomavirus (HPV). Which intervention should the nurse implement?

Reinforce the importance of annual papanicolaou (Pap) smears.

To evaluate the effectiveness of male client's new prescription for ezetimibe, which action should the clinic nurse implement?

Remind the client to keep his appointments to have his cholesterol level checked.

Suicide precautions are initiated for a child admitted to the mental health unit following an intentional narcotic overdose. After a visitor leaves, the nurse finds a package of cigarettes in the client's room. Which intervention is most important for the nurse to implement?

Remove cigarettes for the client's room A family member of a frail elderly adult asks the nurse about eligibility requirements for hospice care.

At 1615, prior to ambulating a postoperative client for the first time, the nurse reviews the client's medical record. Based on date contained in the record, what action should the nurse take before assisting the client with ambulation:

Remove sequential compression devices. Apply PRN oxygen per nasal cannula. Administer a PRN dose of an antipyretic. Reinforce the surgical wound dressing. Rationale: Sequential compression devices should be removed prior to ambulation and there is no indication that this action is contraindicated. The client's oxygen saturation levels have been within normal limits for the previous four hours, so supplemental oxygen is not warranted.

The nurse inserts an indwelling urinary catheter as seen in the video what action should the nurse take next?

Remove the catheter and insert into urethral opening Observe for urine flow and then inflate the balloon. Insert the catheter further and observe for discomfort. Leave the catheter in place and obtain a sterile catheter. Rationale: the catheter is in the vaginal opening.

The leg of a client who is receiving hospice care have become mottled in appearance. When the nurse observes the unlicensed assistive personal (UAP) place a heating pad on the mottled areas, what action should the nurse take?

Remove the heating pads and place a soft blanket over the client's leg and feet. Advise the UAP to observe the client's skin while the heating pads are in place. Elevate the client's feet on a pillow and monitor the client's pedal pulses frequently. Instruct the UAP to reposition the heating pads to the sides of the legs and feet.

During an annual physical examination, an older woman's fasting blood sugar (FBS) is determined to be 140 mg/dl or 7.8 mmol/L (SI). Which additional finding obtained during a follow-up visit 2 weeks later is most indicative that the client has diabetes mellitus (DM)?

Repeated fasting blood sugar (FBS) is 132 mg/dl or 7.4 mmol/L (SI).

A client with a history of dementia has become increasingly confused at night and is picking at an abdominal surgical dressing and the tape securing the intravenous (IV) line. The abdominal dressing is no longer occlusive, and the IV insertion site is pink. What intervention should the nurse implement?

Replace the IV site with a smaller gauge. Redress the abdominal incision Leave the lights on in the room at night. Apply soft bilateral wrist restraints. Rationale: The abdominal incision should be redressed using aseptic-techniques. The IV site should be assessed to ensure that it has not been dislodged and a dressing reapplied, if need it. Leaving the light on at night may interfere with the client's sleep and increase confusion. Restraints are not indicated and should only be used as a last resort to keep client from self-harm.

When preparing a client for discharge from the hospital following a cystectomy and a urinary diversion to treat bladder cancer, which instruction is most important for the nurse to include in the client's discharge teaching plan?

Report any signs of cloudy urine output.

An older adult male who had an abdominal cholecystectomy has become increasingly confused and disoriented over the past 24 hours. He is found wandering into another client's room and is return to his room by the unlicensed assistive personnel (UAP). What actions should the nurse take? (Select all that apply).

Report mental status change to the healthcare provider. Assess the client's breath sounds and oxygen saturation. Review the client's most recent serum electrolyte values. Rationale: The healthcare provider should be informed of changes in the client's condition (B) because this behavior may indicate a postoperative complication. Diminished oxygenation (C) and electrolyte imbalance (E) may cause increased confusion in the older adult. Raising all four bed rails (A) may lead to further injury if the client climbs over the rails and falls and restrains should not be applied until other measures such as re-orientation are implemented. The nurse should assess the client's increased risk for falls, rather than assigning this to the UAP (D).

An older adult female admitted to the intensive care unit (ICU) with a possible stroke is intubated with ventilator setting of tidal volume 600, PlO2 40%, and respiratory rate of 12 breaths/minute. The arterial blood gas (ABG) results after intubation are PH 7.31. PaCO2 60, PaO2 104, SPO2 98%, HCO3 23. To normalize the client's ABG finding, which action is required?

Report the results to the healthcare provider. Increase ventilator rate. Administer a dose of sodium carbonate. Decrease the flow rate of oxygen. Rationale: This client is experience respiratory acidosis. Increasing the ventilator rate depletes CO2 a, which returns the PH toward normal. Report findings is important but only after increasing ventilator rate.

During discharge teaching, the nurse discusses the parameters for weight monitoring with a client who was recently diagnosed with heart failure (HF). Which information is most important for the client to acknowledge?

Report weight gain of 2 pounds (0.9kg) in 24 hours

A child is admitted to the pediatric unit diagnosed with sickle cell crisis. When the nurse walks into the room, the unlicensed assistive personnel (UAP) is encouraging the child to stay in bed in the supine position. Which action should the nurse implement?

Reposition the client with the head of the bed elevated.

The nurse is preparing a 4-day-old I infant with a serum bilirubin level of 19 mg/dl (325 micromol/L) for discharge from the hospital. When teaching the parents about home phototherapy, which instruction should the nurse include in the discharge teaching plan?

Reposition the infant every 2 hours. Rational: An infant, who is receiving phototherapy for hyperbilirubinemia, should be repositioned every two hours. The position changes ensure that the phototherapy lights reach all of the body surface areas. Bathing, feedings, and diaper changes are ways for the parents to bond with the infant, and can occur away from the treatment. Feedings need to occur more frequently than every 4 hours to prevent dehydration. The infant should wear only a diaper so that the skin is exposed to the phototherapy

The nurse enters a client's room and observes the client's wrist restraint secured as seen in the picture. What action should the nurse take?

Reposition the restraint tie onto the bedframe.

A client who had a below the knee amputation is experiencing severe phantom limb pain (PLP) and ask the nurse if mirror therapy will make the pain stop. Which response by the nurse is likely to be most helpful?

Research indicates that mirror therapy is effective in reducing phantom limb pain You can try mirror therapy, but do not expect to complete elimination of the pain Transcutaneous electrical nerve stimulators (TENS) have been found to be more effective Where did you learn about the use of mirror therapy in treating in treating phantom limb pain? Rationale: pain relief associated with mirror therapy may be due to the activation of neurons in the hemisphere of the brain that is contralateral to the amputated limb when visual input reduces the activity of systems that perceive protopathic pain.

During shift report, the central electrocardiogram (EKG) monitoring system alarms. Which client alarm should the nurse investigate first?

Respiratory apnea of 30 seconds In early septic shock states, what is the primary cause of hypotension? Peripheral vasoconstriction Peripheral vasodilation Cardiac failure A vagal response Rationale: Toxins released by bacteria in septic shock create massive peripheral vasodilation and increase microvascular permeability at the site of the bacterial invasion.

When development a teaching plan for a client newly diagnosed type 1 diabetes, the nurse should explain that an increase thirst is an early sing of diabetes ketoacidosis (DKA), which action should the nurse instruct the client to implement if this sign of DKA occur?

Resume normal physical activity Drink electrolyte fluid replacement Give a dose of regular insulin per sliding scale Measure urinary output over 24 hours. Rationale: As hyperglycemia persist, ketone body become a fuel source, and the client manifest early signs of DKA that include excessive thirst, frequent urination, headache, nausea and vomiting. Which result in dehydration and loss of electrolyte. The client should determine fingersticks glucose level and self-administer a dose of regular insulin per sliding scale.

Fallowing an outbreak of measles involving 5 students in an elementary school, which action is most important for the school nurse to take?

Review the immunization records of all children in the elementary school Report the measles outbreak to all community health organizations Schedule a mobile public health vehicle to offer measles inoculations to unvaccinated children. Restrict unvaccinated children from attending school until measles outbreak is resolved.

The healthcare provider prescribes a low-fiber diet for a client with ulcerative colitis. Which food selection would indicate to the nurse the client understands they prescribed diet?

Roasted turkey canned vegetables. Rationale: Foods allowed on a low-fiber diet includes roasted or baked turkey and canned vegetables the foods in the other options are not low in fiber.

The nurse is preparing to administer 1.6 ml of medication IM to a 4 month old infant. Which action should the nurse include?

Select a 22 gauge 1 1⁄2 inch (3.8 cm) needle for the intramuscular injection Administer into the deltoid muscle while the parent holds the infant securely Divide the medication into two injection with volumes under 1ml Use a quick dart-like motion to inject into the dorsogluteal site. Rationale: IM injection for children under 3 of age should not exceed 1ml. divide the dose into smaller volumes for injection in two different sites.

A male client receives a thrombolytic medication following a myocardial infarction. When the client has a bowel movement, what action should the nurse implement?

Send stool sample to the lab for a guaiac test Observe stool for a day-colored appearance. Obtain specimen for culture and sensitivity analysis Asses for fatty yellow streaks in the client's stool. Rationale: Thrombolytic drugs increase the tendency for bleeding. So guaiac (occult blood test) test of the stool should be evaluated to detect bleeding in the intestinal tract.

A client refuses to ambulate, reporting abdominal discomfort and bloating caused by "too much gas buildup" the client's abdomen is distended. Which prescribed PRN medication should the nurse administer?

Simethicone (Mylicon)

A nurse-manager is preparing the curricula for a class for charge nurses. A staffing formula based on what data ensures quality client care and is most cost-effective?

Skills of staff and client acuity

The nurse is caring for a 4-year-old male child who becomes unresponsive as his heart rate decreases to 40 beats/minute. His blood pressure is 88/70 mmHg, and his oxygen saturation is 70% while receiving 100% oxygen by non-rebreather face mask. In what sequence, from first to last, should the nurse implement these actions? (Place the first action on top and last action on the bottom.)

Start chest compressions with assisted manual ventilations Administer epinephrine 0.01 mg/kg intraosseous (IO) Apply pads and prepare for transthoracic pacing Review the possible underlying causes for bradycardia

A client who was admitted yesterday with severe dehydration is complaining of pain a 24 gauge IV with normal saline is infusing at a rate of 150 ml/hour. Which intervention should the nurse implement first?

Stop the normal saline infusion.

While visiting a female client who has heart failure (HF) and osteoarthritis, the home health nurse determines that the client is having more difficulty getting in and out of the bed than she did previously. Which action should the nurse implement first?

Submit a referral for an evaluation by a physical therapist.

Which assessment finding for a client who is experiencing pontine myelinolysis should the nurse report to the healthcare provider?

Sudden dysphagia Blurred visual field Gradual weakness Profuse diarrhea

The nursing staff on a medical unit includes a registered nurse (RN), practical nurse (PN), and an unlicensed assistive personnel (UAP). Which task should the charge nurse assign to the RN?

Supervise a newly hired graduate nurse during an admission assessment.

The nurse is assisting a new mother with infant feeding. Which information should the nurse provide that is most likely to result in a decrease milk supply for the mother who is breastfeeding?

Supplemental feedings with formula Maternal diet high in protein Maternal intake of increased oral fluid Breastfeeding every 2 or 3 hours. Rationale: Infant sucking at the breast increases prolactin release and proceeds a feedback mechanism for the production of milk, the nurse should explain that supplemental bottle formula feeding minimizes the infant's time at the breast and decreases milk supply. B promotes milk production and healing after delivery. C support milk production. C is recommended routine for breast feeding that promote adequate milk supply.

The nurse is caring for a group of clients with the help of a licensed practical nurse (LPN) and an experienced unlicensed assistive personnel (UAP). Which procedures can the nurse delegate to the UAP? (Select all that apply)

Take postoperative vital signs for a client who has an epidual following knee arthroplasty Collect a sputum specimen for a client with a fever of unknown origin Ambulate a client who had a femoral-popliteal bypass graft yesterday

A client with muscular dystrophy is concerned about becoming totally dependent and is reluctant to call the nurse to assist with activities of daily living (ADLs). To achieve maximum mobility and independence, which intervention is most important for the nurse to include in the client's plan of care?

Teach family proper range of motion exercises.

One year after being discharged from the burn trauma unit, a client with a history of 40% full-thickness burns is admitted with bone pain and muscle weakness. Which intervention should the nurse include in the clients plan of care?

Teach need for dietary and supplementary vitamin D3. Rationale: Burn injury results in the acute loss of bone as well as the development of progressive vitamin D deficiency because burn scar tissue and adjacent normal-appearing skin cannot convert normal quantities of the precursors for vitamin D3 that is synthesized from ultraviolet sun rays which is needed for strong bones. Clients with a history of full thickness burns should increase their dietary resources of vitamin D and supplemental D3 (B). range of motion (A) and muscle strengthening exercises (D) do not treat he underlying causes of the bone pain and weakness unprotected sunlight (C) should be avoided.

A client who recently underwear a tracheostomy is being prepared for discharge to home. Which instructions is most important for the nurse to include in the discharge plan?

Teach tracheal suctioning techniques

A 350-bed acute care hospital declares an internal disaster because the emergency generators malfunctioned during a city-wide power failure. The UAPs working on a general medical unit ask the charge nurse what they should do first. What instruction should the charge nurse provide to these UAPs?

Tell all their assigned clients to stay in their rooms.

The fire alarm goes off while the charge nurse is receiving the shift report. What action should the charge nurse implement first?

Tell the staff to keep all clients and visitors in the client rooms with the doors closed

The RN is assigned to care for four surgical clients. After receiving report, which client should the nurse see first?

The client who is Three days postoperative colon resection receiving transfusion of packed RBCs.

A client with a prescription for "do not resuscitate" (DNR) begins to manifest signs of impending death. After notifying the family of the client's status, what priority action should the nurse implement?

The client's need for pain medication should be determined.

The nurse performs a prescribed neurological check at the beginning of the shift on a client who was admitted to the hospital with a subarachnoid brain attack (stroke). The client's Glasgow Coma Scale (GCS) score is 9. What information is most important for the nurse to determine?

The client's previous GCS score When the client's stroke symptoms started If the client is oriented to time The client's blood pressure and respiration rate Rationale: The normal GCS is 15, and it is most important for the nurse to determine if it abnormal score a sign of improvement or a deterioration in the client's condition

The nurse is assessing an older adult with type 2 diabetes mellitus. Which assessment finding indicates that the client understands long- term control of diabetes?

The fating blood sugar was 120 mg/dl this morning. Urine ketones have been negative for the past 6 months The hemoglobin A1C was 6.5g/100 ml last week No diabetic ketoacidosis has occurred in 6 months. Rationale: A hemoglobin A1C level reflects he average blood sugar the client had over the previous 2 to 3 month, and level of 6.5 g/100 ml suggest that the client understand long-term diabetes control. Normal value in a diabetic patient is up to 6.5 g/100 ml.

If the nurse is initiating IV fluid replacement for a child who has dry, sticky mucous membranes, flushed skin, and fever of 103.6 F. Laboratory finding indicate that the child has a sodium concentration of 156 mEq/L. What physiologic mechanism contributes to this finding?

The intravenous fluid replacement contains a hypertonic solution of sodium chloride Urinary and Gastrointestinal fluid loss reduce blood viscosity and stimulate thirst Insensible loss of body fluids contributes to the hemoconcentration of serum solutes Hypothalamic resetting of core body temperature causes vasodilation to reduce body heat Rationale: Fever causes insensible fluid loss, which contribute to fluid volume and results in hemoconcentration of sodium (serum sodium greater than 150 mEq/L). Dehydration, which is manifested by dry, sticky mucous membranes, and flushed skin, is often managed by replacing lost fluids and electrolytes with IV fluids that contain varying concentration of sodium chloride. Although other options are consistent with fluid volume deficit, the physiologic response of hypernatremia is explained by hem concentration.

An elderly client with degenerative joint disease asks if she should use the rubber jar openers that are available. The nurse's response should be based on which information about assistive devices?

They decrease the risk for joint trauma

A client is admitted to a medical unit with the diagnosis of gastritis and chronic heavy alcohol abuse. What should the nurse administered to prevent the development of Wernicke's syndrome?

Thiamine (Vitamin B1). Rationale: Thiamine replacement is critical in preventing the onset of Wernickes encephalopathy, an acute triad of confusion, ataxia, and abnormal extraocular movements, such as nystagmus related to excessive alcohol abuse. Other medications are not indicated.

A mother brings her 4-month-old son to the clinic with a quarter taped over his umbilicus, and tells the nurse the quarter is supposed to fix her child's hernia. Which explanations should the nurse provide?

This hernia is a normal variation that resolves without treatment. Rational: an umbilical hernia is a normal variation in infants that occurs due to an incomplete fusion of the abdominal musculature through the umbilical ring that usually resolves spontaneously as the child learns to walk. Other choices are ineffective and unnecessary.

A client with a history of diabetes and coronary artery disease is admitted with shortness of breath, anxiety, and confusion. The client's blood pressure is 80/60 mmHg, heart rate 120 beats/minute with audible third and fourth heart sounds, and bibasilar crackles. The client's average urinary output is 5 ml/hour. Normal saline is infusing at 124 ml/hour with a secondary infusion of dopamine at mcg/kg/minute per infusion pump. With intervention should the nurse implement?

Titrate the dopamine infusion to raise the BP. Rationale: the client is experiencing cardiogenic shock and requires titration per protocol of the vasoactive secondary infusion, dopamine, to increase the blood pressure. Low hourly urine output is due to shock and does not indicate a need for catheter irrigation. Pacing is not indicated based on the client's capillary blood glucose should be monitored, but is not directly indicated at this time.

A client with eczema is experiencing severe pruritus. Which PRN prescriptions should the nurse administer? (Select all that apply)

Topical corticosteroid. Oral antihistamine. Rationale: anti-inflammatory actions of topical corticosteroids and oral antihistamines provide relief from severe pruritus (itching). Other options are not indicated.

A client who is admitted to the intensive care unit with a right chest tube attached to a THORA-SEAL chest drainage unit becomes increasingly anxious and complain of difficulty breathing. The nurse determine the client is tachypneic with absent breath sounds in the client's right lungs fields. Which additional finding indicates that the client has developed a tension pneumothorax?

Tracheal deviation toward the left lung. Rationale: Tracheal deviation toward the unaffected left lung with absent breath sounds over the affected right lung are classic late signs of a tension pneumothorax.

A client in the postanesthesia care unit (PACU) has an eight (8) on the Aldrete postanesthesia scoring system. What intervention should nurse implement?

Transfer the client to the surgical floor.

Following and gunshot wound, an adult client a hemoglobin level of 4 grams/dl (40 mmol/L SI). The nurse prepares to administer a unit of blood for an emergency transfusion. The client has AB negative blood type and the blood bank sends a unit of type A Rh negative, reporting that there is not type AB negative blood currently available. Which intervention should the nurse implement?

Transfuse Type A negative blood until type AB negative is available. Rationale: those who have type AB blood are considered universal recipients using A or B blood types that is the same Rh factor. The client's hemoglobin is critically low and the client should receive a unit of blood that is type A, which must be Rh negative blood. Other options are not indicated in this situation.

A 4-year-old with acute lymphocytic leukemia (ALL) is receiving a chemotherapy (CT) protocol that includes methotrexate (Mexate, Trexal, MIX), an antimetabolite. Which information should the nurse provide the parents about caring for their child?

Use sunblock or protective clothing when outdoors.

A client with history of bilateral adrenalectomy is admitted with a week, irregular pulse, and hypotension. Which assessment finding warrants immediate intervention by the nurse?

Ventricular arrhythmias. Rationale: adrenal crisis, a potential complication of bilateral adrenalectomy, results in the loss of mineralocorticoids and sodium excretions that is characterized by hyponatremia, hyperkalemia, dehydration, and hypotension. Ventricular arrhythmias are life threatening and required immediate intervention to correct critical potassium levels.

A client who is experiencing musculoskeletal pain receives a prescription for ketorolac 15mg IM q6 hours. The medication is depended in a 39mg/ml pre-filled syringe. Which action should the nurse implement when giving the medication?

Waste 0.5 ml from the pre-filled syringe and inject the medication in the ventrogluteal site. Rational: The pre-filled contain 30mg /1ml, so 0.5ml should be wasted to obtain the correct dosage of 15mg for administration in the preferred IM ventrogluteal site. The nurse is responsible for calculating and preparing the prescribed dose using the available concentration, so other options are not indicated.

When teaching a group of school-age children how to reduce the risk of Lyme disease which instruction should the camp nurse include?

Wear long sleeves and pants. Rationale: Lyme disease is it tick bone disorder and is transmitted to a child via a tick bite. Keeping the skin covered reduces the risk of being bitten by a tick. Other options are not reduce the risk for tick bites

A client is being discharged home after being treated for heart failure (HF). What instruction should the nurse include in this client's discharge teaching plan?

Weigh every morning Eat a high protein diet Perform range of motion exercises Limit fluid intake to 1,500 ml daily

The nurse is managing the care of a client with Cushing's syndrome. Which interventions should the nurse delegate to the unlicensed assistive personnel (UAP)? (Select all that apply)

Weigh the client and report any weight gain. Report any client complaint of pain or discomfort. Note and report the client's food and liquid intake during meals and snacks.

When planning care for a client with acute pancreatitis, which nursing intervention has the highest priority?

Withhold food and fluid intake. Rational: The pathophysiologic processes in acute pancreatitis result from oral fluid and ingestion that causes secretion of pancreatic enzymes, which destroy ductal tissue and pancreatic cells, resulting in auto digestion and fibrosis of the pancreas. The main focus of the nursing care is reducing pain caused by pancreatic destruction through interventions that decrease GI activity, such as keeping the client NPO. Other choices are also important intervention but are secondary to pain management.

The nurse is developing an educational program for older clients who are being discharged with new antihypertensive medications. The nurse should ensure that the educational materials include which characteristics? Select all that apply

Written at a twelfth grade reading level Contains a list with definitions of unfamiliar terms Uses common words with few Syllables Printed using a 12 point type font Uses pictures to help illustrate complex ideas Rationale: During the aging process older clients often experience sensory or cognitive changes, such as decreased visual or hearing acuity, slower thought or reasoning processes, and shorter attention span. Materials for this age group should include at least of terms, such as a medical terminology that incline may not know and use common words that expresses information clearly and simply. Simple, attractive pictures help hold the learner's attention. The reading level of material should be at the 4th to 5th grade level. Materials should be printed using large font (18-point or higher), not the standard 12-point font.

The healthcare provider prescribes the antibiotic Cefdinir (cephalosporin) 300mg PO every 12 h for a client with postoperative wound infections. Which feeds should the nurse encourage this client to eat?

Yogurt and/or buttermilk. Avocados and cheese Green leafy vegetables Fresh fruits

The nurse is assessing a client's nailbeds. Witch appearance indicates further follow-up is needed for problems associated with chronic hypoxia?

clubbing

A preeclamptic client who delivered 24h ago remains in the labor and delivery recovery room. She continues to receive magnesium sulfate at 2 grams per hour. Her total input is limited to 125 ml per hour, and her urinary output for the last hour was 850 ml. What intervention should the nurse implement?

discontinue the magnesium sulfate immediately Decrease the client's iv rate to 50 ml per hour Continue with the plan of care for this client Change the client's to NPO status Rationale: continue with the plan. Diuresis in 24 to 48h after birth is a sign of improvement in the preeclamptic client. As relaxation of arteriolar spasms occurs, kidney perfusion increases. With improvement perfusion, fluid is drawn into the intravascular bed from the interstitial tissue and then cleared by the kidneys


Conjuntos de estudio relacionados

Intermediate Accounting Exam 1 (Chapter 1-4)

View Set

Geometry, Chapter 2, Algebraic Properties

View Set

Chapter 26 Quiz - Soft-Tissue Injuries

View Set

block 09 Landing Gear Systems Quiz 1

View Set